پزشکی و سلامتآناتومی و فیزیولوژیبیماری‌هاصدمات و آسیب‌دیدگی‌ها

پاورپوینت مناسب دانشجویان رزیدنت (آزمون بالینی ساختارمند عینی)

صفحه 1:
Pulmonology: OSCE & PMP

صفحه 2:
خانم 48 ساله که 3 ساعت پس از سقوط از پله منزل دچار تگی نفس و هذیان گویی شده و در بررسی های اولیه شواهد بالا به دست آمده است؟

صفحه 3:
48-YEAR-OLD MAN WHO HAD SUSTAINED A TRAUMATIC FRACTURE OF the left femoral shaft in a motorcycle accident 20 hours earlier was referred for a deterioration of consciousness that began after the accident. He was unresponsive to verbal stimuli, and no focal neurologic abnormalities were found on examination. An arterial blood gas analysis revealed mild hypoxemia, but a ra- diograph of the chest was normal. Tracheal intubation was performed for airway protection. Magnetic resonance imaging of the brain revealed multiple hyperin- tense punctate lesions disseminated throughout the cerebral white matter on T,- weighted axial images (Panel A) and a so-called starfield pattern on diffusion- weighted images (Panel B). Seven hours after presentation, widespread petechiae developed on the chest, upper arm, and armpit (Panel C). The combination of a recent fracture of a long bone, petechial rash, mild hypoxemia, and profound im- pairment of consciousness prompted a diagnosis of fat embolism syndrome. The patient recuperated gradually after orthopedic repair of the fracture and rehabilita- tion therapy.

صفحه 4:
Ficune 2A 1Syearokd mam at 2 days after & ttn Top, HRGT son hn a nes gm reveal 1 predominantly peripheral ‏یاس اه ساسا‎ Opectiestsneated with smooth ard nodhlar cepa theketing ‘oan, B: HRC obtained at lower level shows retve spring Of some secondary lobules, Findings i the paint ore mar ponomced Hin the ebscrved in the patent in Figure 1 Ficune 4. A 27-year-old man with hyposemia and buccal petechiae 3 days aller injury. Top, A: HCE obtained just below the tracheal hifureation reveals a predominantly nodular pattern Note that bronchowaseular bundles are thin and smooth. Bottom, B:TIRCT obtained at the level of pulimonary veins shows similar pattem and severity of findings

صفحه 5:
CASE 1: A 65 y,o man with a history of 40 p/y smoking presented because of 4 Ws of progressive exertional dyspnea and non productive cough!!!

صفحه 6:
CASE 3: a 67 y,oman with progressive 7 2 since 2 y ۱ Figure 6.3. HRCT scan in early phase of IPF shows predominantly subpleural reticular pattem (a), and in the late phase predominantly subpleural coarse reticular pattem, septal thick- ening, traction bronchiectasis, and honeycombing. The ground-glass opacities are usually limited, and if extensive (>30% of the lung involved) an alternative diagnosis should be con- sidered. This particular 70-year old patient experienced progressive dyspnea during lst 8 months, quitted smoking ost weight and was examined for suspected IPF. As he fulfilled the major and minor criteria for IP, including the HRCT finding, the open lung biopsy was not considered necessary. As the disease progressed the corticosteroids and azathioprine were applied with insignificant efficacy (b).

صفحه 7:
CASE 4: A 40 y,o nonsmoker woman with progressive dyspnea of 4 m duration accompanied with fever, elevated LDH and 3

صفحه 8:

صفحه 9:
CASE 5: A 85 y,o man with acute progressive shortness of breath since 48 h ago

صفحه 10:
CASE 6: A 81/Y,0 MAN WITH FLU-LIKE SYMPTOMES SINCE 3 WEEKS AGO + ۳۹ ‏ام‎ DYSENEA ae microscopic views of COP. “./. Note the absence of old » fibrosis or architectural 1g appearance & In tissue plugs inflammatory infiltrate. something that is

صفحه 11:
هر دو بیمار آقای 57 ساله سیگاری 30 0/۷ با شکایت سرفه و تنگی نفس از 5 ماه قبل صوق و كفيس درمان؟؟ Langerhans cell histiocytosis. (A) The characteristic combination of thin-walled cysts and poorly defined nodules, some of which are just beginning to cavitate.* (B) Image from a patient with more advanced disease. There are numerous irregularly shaped cysts bilaterally.

صفحه 12:

صفحه 13:
‎cunt 9.62‏ امعم ‎Diffuse bilateral thin-walled cysts throughout the lungs in women of childbearing age are diagnostic of LAM. ‎Extrathoracic related findings are present in >70% of cases. ‎

صفحه 14:
خانم ۳۴ ساله با علایم تب و لرز و میالژی از ۱۴ روز قبل که طی ۱۲ ساعت گذشته دچار تنگی نفس پیشرونده و هیپوکسی شدید و سیانوز شده است

صفحه 15:
۱۱۱۱3۸ 0۱۸6۱05۲16 8118184 ARDS Chest Absence of Left |Severity: Oxygenation _Onset_Radiograph___Atrial Hypertension _ Mild: 200 mmHg Acute Bilateral alveolar. PCWP <18 mmHg or < Pao /Fia, < 300 mmHg or interstitial no clinical evidence Moderate: 100 mmHg infiltrates of increased left atrial < Pao,/Fio, > 200 mmHg pressure Severe: Pao_/Fio, > 100 mmHg Abbreviations: ARDS, acute respiratory distress syndrome; Fio,, inspired O, percentage; ‏يدوم‎ arterial partial pressure of O,; PCWP, pulmonary capillary wedge pressure.

صفحه 16:
InrriaL MANAGEMENT OF ARDS LUTEP739 EVIDENCE-BASED RECOMMENDATIONS FOR ARDS THERAPIES: [Goaisendtimis: == —satment__Recommendation’ _ Thumalpressuesiied > Wal vokme <6 mig PEW eee A 6 Plateau pressure < 30 omHO lume = RR < 35 bpm Minimized left atria filing pressures 8 High PEEP or‘open lung’ c ‏ديمع‎ Prone position c PEEP > 10 ‏ماني‎ Recruitment maneuvers c ees High-ftequency ventiation D ECMO ۴ 557 Early neuromuscular blockade A Minimize acidosis AR < 35 bpm Glucocorticoid treatment D Surfactant replacement, Inhaled NO, Inhaled D epoprostendl, and other antinflammatory 7 therapy (eg, ketoconazole PGE!, NSAIDs) 265 mmHg “key:A recommended thetapy bated on ston cna evidence fom randomized ‏رد رم‎ nical tials; 8, recommended therapy based on supportive but limited clinical data; C, recommended only as alternative therapy on the basis of indeterminate evidence, D, not recommended on the bass of clinical evidence against efficacy of therapy, IGURE 322-5 Algorithm for the initial management of ARDS. Clinical trials have provided evidence-based therapeutic goals fora ‏و ی‎ nic cade NeAlos nowsetol airman dae PEEP stepwise approach to the early mechanical ventilation, oxygenation, positive end-expiratory pressure, PCE, prostaglandin E and correction of acidosis and diuresis of criticaly ill patients with ARDS. Fo, inspired O, percentage; MAP, mean arterial pressure; PBW, predicted body weight; PEEP, positive end expiratory pressure; RR, respiratory rate; SpO,, arterial oxyhemoglobin saturation measured by pulse oximetry. 7

صفحه 17:
خانم جوان با شکایت درد قفسه سینه از صبح روز مراجعه با انتشار به بازوی راست و تنگی نفس. بیمار سابقه فشار خون و 6۴۲ و سرفه مزمن را نیز داشته است.

صفحه 18:
PNEUMOMEDIASTINUM In this condition, there is(gas in the interstices of the mediastinum. The three main causes are (1) alveolar rupture with dissection of air into the mediastinum; (2) perforation or rupture of the esophagus, trachea, or main bronchi; and (3) dissection of air from the neck or the abdomen into the mediastinum. Typically, there is severe subster- nal chest pain with or without radiation into the neck and arms. The physical examination usually reveals subcutaneous emphysema in the suprasternal notch and! Hamman’s sign, which is a crunching or click- ing noise synchronous with the heartbeat and i is best heard in the left ¢ rmed with the chest ph. will be absorbed faster if the patient inspires high ‘concentrations of oxygen. If mediastinal structures are compressed, the compression can erelieved with needle aspiration.

صفحه 19:
نام دو دستگاه مشاهده شده را بگویید و برای هر مورد یک مثال دارویی بزنید:

صفحه 20:
o% i 3 MDI with Spacer ‏املا‎ Diskus Handihaler Twisthaler_ Autohaler 3 > ١ BI ۱ ‏سم‎ 1 © Aerolizer Flexhaler Neohaler Respimat Pressair Turbuhaler Ellipta

صفحه 21:
301606066066156 76 25mg/2 Smt nebule smg/2smLreaiie ‘stata ‘SALGUTAMOL WITH IPRATROPIUM Nebuter olson saibuera 25 me + ‘erro 500 meg 2.5 mL ‘NEDOCROWIL Acoso whaler ‏لد‎ ‘Te Subsidised Inhalers for Asthma or COPD New Zealand March 2016 - ‏سس‎ rr IPRATROPIUM Aerosol inhaler 5 8 en Aerosol inhaler ‏و سیم‎ ei see wal ee ۳ ‏هه جع‎ ‏ص امت‎ e = =a & = = = ‘Mast Cell Stabilisers ‏سس‎ nen ‏اند‎ SEER Areal ae ae covet ‏عمج‎ ee SABA —Short-Acting Beta2 Agonists ne awn 0 a f = 2 ‏ع ع‎ SAIRUTAMOL Aerosol nher| ‏هوم مهد‎

صفحه 22:
Table 1: Guide to addition of therapies* Lapay Ics) “ed boves win crosses neste cases of topes tat should not be used togeter SABA SAMA LAMA LABA AEX IKEA مجعم ,ممم "ماده مدای ۱ ipratropium bromide (Atrovent) ‘Totrepium bromige (Spiiva™) {lycopyrrenium bromide (Secbr™) mediate (ineuse™). Saimeterol (Serevent) وعم سحي . ‎Forale™)‏ اس ام 0 ا ‎bromide (URibro”) Gree)‏ {+ Futeasone propirata/Saimeterl (See) py scaone = 1 Busesonice/efarmoreal (Symbenrt™) Fh furoateyviantere (Bre0™) ‘Acidnium bromide (Bretarie™) Comes sana ann aa casa Bat! Ics, 4 td

صفحه 23:
45 ساله با تتگی نفس و هموپتزی مراجعه نموده است. با توجه به تگرافی بیمار 5 تشخیص افتراقی مطرح نمایید:

صفحه 24:
| = infection pulmonary abscess TB, fungal, staph aureus T = trauma Pneumatocoeles Y = youth CPAM pulmonary sequestration bronchogenic cyst CAVITY mnemonic : C = cancer bronchogenic carcinoma : most frequently SCC cavitatory metastasis : again most frequently SCC A = autoimmune granulomas Wegener's granulomatosis rheumatoid arthritis (rheumatoid nodules) V = vascular pulmonary embolus / pulmonary infarction

صفحه 25:
خانم ۵۶ساله با درد مفاصل از ۴سال قبل و خشکی صبحگاهی در حد ۳۰دقیقه كه از "ماه قبل دچار تنگی نفس شده است . ۱.توصیف؟ ۲.تشخیص؟؟

صفحه 26:
اور یت وزیا Pleural effusion or thickening Interstitial fibrosis (most frequently usual interstitial pneumonia type) Constrictive obliterative bronchiolitis a Bronchiectasis Organizing pneumonia Follicular bronchiolitis Drug-induced lung disease (methotrexate) | Necrobiotic nodules/Caplan’s syndrome ’ ۱ Rheumatoid disease. Two cavitating necrobiotic nodules are visible*

صفحه 27:

صفحه 28:
Differential Diagnosis ¢ Malignancy * Granulomatous disease ¢ Inflammation ¢ Benign neoplasm * Congenital multiple nodules on left (all less than 3 cm) and nodules and one mass on the right (3.4 cm)

صفحه 29:
خانم 44 ساله با شکایت تنگی نفس مراجعه نموده و به علت رویت پلورال افیوژن تحت آسپیراسیون مایع پلور قرار گرفته است, تب نداشته و مایع بدبو تق فرص 8 3 علت اصلی ایجاد این حالت؟

صفحه 30:
Lymphomas Bronchogenic carcinoma Other solid tumors” Surgical (especially thoracic surgery) Non surgical trauma Congenital Other unidentifiable causes Sarcoidosis! Congestive cardiac failure Lymphengioleiomyomatosis (LAM) Lymphengiomatosis Yellow nail syndrome Gorham’s syndrome Castleman’s disease Superior vena cava obstruction Kaposi sarcoma in acquired immunodeficieney syndrome (ATDS) Radiation therapy to the mediastinum Tuberculosis, histoplasmosis, or filariasis Malignancy Trauma Idiopathic Miscellaneous Infections Chylothorax A chylothorax occurs when the thoracic duct is disrupted and chyle accumulates in the pleural space. The most common cause of chylothorax is trauma (most frequently thoracic surgery), but it also may result from tumors in the mediastinum. Patients with chylothorax present with dyspnea, and a large pleural effusion is present on the chest radiograph. Thoracentesis reveals milly fluid, and biochemical analysis reveals a triglyceride level that exceeds 1.2 mmol/l. (110 mg/ dL). Patients with chylothorax and no obvious trauma should have a lymphangiogram and a mediastinal CT scan to assess the mediastinum for lymph nodes. The treatment of choice for most chylothoraxes is insertion of a chest tube plus the administration of octreotide. If these modalities fail, a pleuroperitoneal shunt should be placed unless the patient has chylous ascites. Alternative treatments are ligation of the thoracic duct and percutaneous transabdominal thoracic duct block- age. Patients with chylothoraxes should not undergo prolonged tube thoracostomy with chest tube drainage because this will lead to mal- nutrition and immunologic incompetence.

صفحه 31:
آقای ۶۰ ساله سیگاری با تنگی نفس از ۵ ماه قبل که از ۲ ماه قبل علایم وی تشدید یافته است. توصيف؟ 1 تشخيص؟؟ درمان؟؟؟

صفحه 32:
FIGURE 5-12, Usual interstitial pneumonia (UIP). A, There are coarse rekular ntrstal markings representing fibrosis, seen herein a dose-up of the right lng base (whe crcl). The findings in UP ‘occur predominantly at the lung bases, B, An axal CT scan ofthe chest shows abnormalities atthe lung bases ina subpleural location, the typical dstnbution for UP. These are small estic spaces called fon ‘excombing (whe arrows) with evidence of bronchiectass a5 manifest by thickened bronchial wals (dotted white aro)

صفحه 33:
Usual interstitial pneumonia. In the upper lobes anteriorly there are peripheral irregular lines with areas of honeycombing.* Usual interstitial pneumonia. HRCT abnormalities predominate in the posterior, and subpleural regions of the lower lobes and comprise honeycombing and traction bronchiectasis within the abnormal lung.*

صفحه 34:
آقای ۴۴ ساله سیگاری با سرفه و تنگی نفس فعالیتی از ۳ ماه قبل: توص یف #تشخيص و تشخيصن. اقتراف؟؟ ۳. درمان؟؟؟

صفحه 35:
RB-ILD. HRCT shows: (A) subtle areas of ground glass opacification and (B) ill-defined centrilobular nodules * DIP. There is diffuse ground- glass opacification throughout the lungs due to inflammatory > infitrate.!

صفحه 36:
آقای ۳۲ ساله با آسم مقاوم به درمان از ۱ سال قبل که اخیرا حملات وی بسیار شدید شده 7”

صفحه 37:

صفحه 38:
~ f FINGER-IN-GLOVE SIGN

صفحه 39:
خانم ۳۴ ساله با یک نوبت سابقه پنوموتوراکس سال قبل با شکایت تنگی نفس ناگهانی و سی تی اسکن زیر به شما مراجعه نموده است. توصیف و تشخیص و درمان؟؟؟

صفحه 40:
Treatment Progression is common, with a median survival of 8-10 years from diagnosis. No therapy is of proven benefit in LAM. Sirolimus, an inhibitor of the mammalian target of rapamycin (mTOR), appears to be an active agent for LAM. After 12 months, it stabilized ung function (FVG, FEV, and functional residual capacity) and was associated with a reduction in symptoms and improvement in quality of life. Adverse effects (c.g., mucositis, diarrhea, nausea, hypercholes- terolemia, acneiform rash, peripheral edema) were more common in the sirolimus group, but serious adverse effects were not increased. Subjects were followed off sirolimus for an additional 12 months, during which time pulmonary function declined at the same rate as in the placebo group. Progesterone and luteinizing hormone-releasing hormone analogues have been used. Oophorectomy is no longer rec- ommended, and estrogen-containing drugs should be discontinued. Lung transplantation offers the only hope for cure despite reports of recurrent disease in the transplanted lung, PULMONARY LYMPHANGIOLEIOMYOMATOSIS i || Manifestations Pulmonary LAM is a rare condition that afflicts premenopausal women and should be suspected in young women with “emphysema,” recurrent pneumothorax, or chylous pleural effusion. It is often misdiagnosed as asthma or chronic obstructive pulmonary disease, Whites are affected much more commonly than are members of other racial groups. The disease accelerates during pregnancy and abates after oophorectomy. Common complaints at presentation are dyspnea, cough, and chest pain, Hemoptysis may be life threatening. Spontaneous pneumothorax occurs in 50% of patients; it may be bilateral and necessitate pleurodesis. Meningioma and renal angiomyolipomas (hamartomas), characteristic findings in the genetic disorder tuberous sclerosis, are also common in patients with LAM. Chylothorax, chyloperitoneum (chylous ascites), chyluria, and chylopericardium are other complications, Pulmonary function testing usually reveals an obstructive or mixed obstructive-restrictive pattern, and gas exchange is often abnormal. HRCT shows thin-walled ‘cysts surrounded by normal lung without zonal predominance, Histologic Findings Pathologically, LAM is characterized by the pro- liferation of atypical pulmonary interstitial smooth muscle and cyst formation. The immature-appearing smooth-muscle cells react with monoclonal antibody HMB45, which recognizes a 100-kDa glycopro- tein (gp100) originally found in human melanoma cells.

صفحه 41:
آقای ۵۳ ساله مسیول تاسیسات بیمارستان که با شکایت تنگی نفس و سرفه های خشک از ۳ ماه قبل مراجعه نموده است. ۱ تشخیص و توصیف؟ ۲ درمان پیشنهادی؟؟

صفحه 42:
Asbestosis Radiographic features Plain radiograph There are no pathognomonic radiological features specific for asbestosis '. Chest radiograph may show irregular opacities with a fine reticular pattern, Additional evidence of asbestos exposure such as calcified or noncalcified pleural plaques may be evident. Histology from a lung biopsy showing asbestos bodies. Ferruginous bodies consisting of asbestos fibers coated by iton-protein-mucopelysaccharde material with Jgolden brown, beaded appearance. Th two longest asbestos bodies a the center ofthe igure are present within a muhinuceated giant cll. Hematoxylin and eosin stain, +400), (Courtesy of Dr. David Hwang, Toronto General Hospital)

صفحه 43:
آقای ۳۳۴ ساله سیگاری با شکایت سرفه» تنگی نفس و کاهش وزن ۱۲ کیلو از ۶ ماه قبل که از ۴ هفته قبل دچار پلی اوری نیز شده است. ۱.توصیف و تشخیص؟ ۲.درمان؟؟

صفحه 44:
These cysts start as round structures but finally coalesce to be- come the typical bizarre shaped cysts of LCH. In patients with LCH 95% have a smoking history. On the left radiological pathological correlation of Langerhans cell histiocytosis in respectively nodular stage and early and late cystic stage

صفحه 45:
خانم ۴۴ ساله با سرفه خلط دار کف آلود از ۲ ماه قبل همراه با ضعف و کاهش وزن و تنگی نفس فعالیتی:

صفحه 46:
‘PULMONARY ALVEOLAR PROTEINOSIS (PAP) inal Manifestations Although not strictly an TLD, PAP resembles ais therefore considered with these condition, It hasbeen p hata defect In macrophage function ability to process surfactant, may play of PAP: acquired (>90% of all case), congenital, and seo ital PAP is transmitted in an ato imal recesive manner and is caused by homorygosty for a frame ‘hifi mutation (121ins2) in the SP-B gene, which leads to an unstable SP-B mRNA, reduced protein levels, and secondary disturbances of SP-C processing. Secondary PAP is rare among adults and is caused by ysnurie protein intolerance, acute alcoss and other inhalato gradromes, immunodeficiency disorders, and malignancies (almost exclusively of hematopoietic origin) and hematopoietic disorders, nonproductive cough 1s common, But occasionally expec toration of “chunky” gelatinous material may occur. Palyythemia hypergammaglobulinemia, and increased LDH level are-common, Markesly elevated serum levels of lung surfactant proteins A and D have been found in PAP. In the absence of any known secondary cause ‘of PAP, an elevated serum anti-GM-CSF ter is highly sensitive and specific forthe diagnosis of acquired PAP. BAL. id levels of ant GM-CSF antibodies correlate bette withthe severity of PAP than do serum titers, Radiographically bilateral symmetric alveolar opacities located centrally in mide and lower lung zones result ina “bat wing bution. HRCT shows a ground-gass opacification and thickened Intralobular structure and interlobular septa. 00 i Findings This difuse dis tion of an amorphous, per is characterized by the accu ic aid-Schif-postive lipopo whole lung lavage tinaceous material inthe distal ar paces. There a litle or no hung inflammation, and the underlying hung architecture preserved (SSE pede ‏وش وت‎

صفحه 47:
آقای ۵۰ ساله پا سابقه ترومای شدید و بستری ۴ ماهه در بخش مراقبتهای ویژه ۳ ‎es =e‏ ع هد 3 2 3 که با شکایت تنقس صدا دار به ‎ve‏ 7 سب 5 8 0 340 33 شما مراجعه نموده است. * 37 0 6 كيه 000 3 5 7 00 3 ات ico (bl) ‏و‎ 6 $0 98 SSPRED = percentage of praised va 3 مگ : ee 5 Expired مسا ۰ و 5 a2 2

صفحه 48:
‘PRED 3 134 148 38 sé 1 3 لم8 ذا عناص ‎inspired‏ 5 37 a oat 340. ۳ ‏کف‎ ‎3 ‎7 ‎838 me av RY/TLC ve FEY, FEV/FVe Mw مت ico (bbs) Spo: SERED pceing: priced ae, Exp flow (Lise) Insp fow Interpretation: Abnormal, Severe fixed airway obstruction is indicated by the reduced FEV, and MVV and shape of the inspiratory and expiratory Bow ‘volume curves. There is no immediate response to bronchodilator. Du.co is mildly reduced, consistent with a pulmonary parenchymal or vascular process. ‎cna‏ عد جاه وه ووه امه سا و

صفحه 49:
Restriction due to Restriction due to. Restriction due to Obstruction Obstruction due to increased lung Chest wall respiratory muscle due to airway decreased clastic recoil ‘abnormality weakness narrowing elastic recoil (pulmonary (moderate (myasthenia (acute (severe fibrosis) obesity) gravis) asthma) emphysema) Te 60% ‏و‎ 73 100% 130% FRC 60% 65% 100% 104% 220% RV 60% 100% 120% 120% 810% Fve 60% 92% 60% 60% 95% pre-b.d. FEV, 750 92% 60% expla Pow 1.0 1.0 1.0 15 ‏ما‎ 60% 9596 20 120% 40% ۳ 3 ۱ x ‘| FIGURE 306e6 Common abnormalities of | ‏ا‎ function (see ‏اس‎ Pulmonary function values are expressed as a percentage of normal predicted values, except for R._, which is expressed as cmH,O/L per sec (normal, <2 cmH,O/L per second). The figures at the bottom of each column show the typical configuration of flow-volume loops in each condition, including the flow-volume relationship during tidal breathing, b.d, bronchodilator; ‏ي‌ه‎ diffusion capacity of lung for carbon monoxide; FEV, forced expiratory volume in 1 sec; FRC, functional residual capacity; FVC, forced vital capacity; R, airways resistance; RV, residual volume; TLC, total lung capacity. al. é __fiow

صفحه 50:
7 ۸ 8 6 D E 5 6 3 5 ‏و‎ ‏لا‎ 2 2 1 2 ‏و‎ ‎Set ‎22 ‎£3 ‎a ‎2 4 £5 6 tf // ۳ ts .5. 7 100 0100 0100 0100 0100 0 Vital capacity (%) Schematic flow-volume loop configurations in a spectrum of airway lesions. A is normal; 8 is variable extrathoracic upper airway obstruction; C is variable intrathoracic upper airway lesion; D is fixed upper airway obstruction; and E is small airway obstruction. L/S = liters per second; = ventilation.

صفحه 51:
Nephrology: OSCE & PMP

صفحه 52:
آقای ۵۷ ساله با شکایت تیره شدن رنگ ادرار و ادم پیش رونده اندام تحتانی. در معاینه اولیه فشار خون بالا داشته و معاینات قلب وپوست وشکم طبیعی است. سمع ریه رال منتشر دارد. بررسی های اولیه به شرح زیر است: ‎aM ۷4‏ ی ات ی ی 5 111 VF. 13 ۷6 RHYTHM STRIP: 11 25 ۳۳/۵۵۵: ۱

صفحه 53:
نوار را تفسیر نموده و ۳ يافته مهم آن را نام ببريد: ‎MAT:‏ The cardinal features are irregularity and a plethora of different P- wave morphologies. You need to have *Tachycardia (HR >100) *Irregular rate “Variability in P wave morphology and PR length

صفحه 54:
هماچوری + پروتینوری + گلوکوزاوی+ ييوردى 3 8 8 Urine Dipsteak: “| DPE 0000 7 60 seclseg "029 1.005 1020, 7 Bo SH co se 00 50 ee 5-06 a 5 ‏و‎ = 0 6275 ca. & Biood/Hemoglobin/ eo 11111 ca. 25 50 80 Nitritenitritonitrtos ۴ 0 Ketones! C.Cetinicos 60 seciseg, Bilirubin/Bilirrubina/ 60 seclseg. Ubilinogen(oy "> Utobitinogénio 160 seciseg. Protein/Proteinas/ Prooinas 00 seen 15 (0.15) 30/0. 100 300) 1000 ‏نا )30:03 )015( سس تسه‎ 0 1000/1) molt (ot) Gicose 60 sex/seg ‘normal 100(&5) 300(17) 1000 55) mg/dl (nmol)

صفحه 55:
Urinary Sediment:

صفحه 56:

صفحه 57:
Diffuse pulmonary haemorrhage with Bilateral GGO

صفحه 58:

صفحه 59:

صفحه 60:
چه درمانی در این مرحله برای ‎slay‏ پیشنهاد میکنید؟؟ target. Between 10 and 15% of sera from patients with Goodpasture’s syndrome also contain ANCA antibodies against myeloperoxidase. This subset of patients has a vasculitis-associated variant, which has a surprisingly good prognosis with treatment. Prognosis at presentation is worse if there are >50% crescents on renal biopsy with advanced fibrosis, if serum creatinine is >5-6 mg/dL, if oliguria is present, or if there is a need for acute dialysis. Although frequently attempted, most of these latter patients will not respond to plasmapheresis and steroids. Patients with advanced renal failure who present with hemoptysis should still be treated for their lung hemorrhage, as it responds to plasmapheresis and can be lifesaving. Treated patients with less severe disease typically respond to 8-10 treatments of plasmapheresis accom- panied by oral prednisone and cyclophosphamide in the first 2 weeks. Kidney transplantation is possible, but because there is risk of recur- rence, experience suggests that patients should wait for 6 months and until serum antibodies are undetectable.

صفحه 61:
Alz2y.o girl with sudden onset of generalized edema and

صفحه 62:
پسر ۲۳ ساله با نارسایی پیشرونده کلیه وهماچوری و شرح حال مثبت خانوادگی از نظر 2580

صفحه 63:
B پسر ۱۵ ساله با هماچوری میکروسکوپی پایدار و پروتینوری خفیف هر کی بگه جایزه داره!

صفحه 64:
: ‏ساله با چندین نوبت بررسى جهت هماچوری میکروسکوپی‎ 52 ila ‏1.توصیف دقیق رادیولوژی‎

صفحه 65:
آقای 54 ساله 255 ‎ESRD‏ تحت دیالیز با سابقه بستری 2 ماه قبل به علت سر درد های راجعه شدید در یک بیمارستان تخصصی اعصاب که از 2 هفته قبل دچار ضایعات پیشرونده پوستی شده. 1.تشخیص 2.علت 3. توصیه های پیشگیرانه احتمالی A rate complication, nephrogenic systemic fibrosis (NSE), has recently been reported in patients with renal insuffi exposed to gadolinium contrast agents, The onset of NSF has and 75 days following exposure; histologic features include thickened collagen bundles with surrounding clefts, mucin deposition, and increased numbers of fibrocytes and clastic fibers in skin. In addition to dermatologic symptoms, other manifestations include widespread fibrosis of the skeletal muscle, bone, lungs, pleura, pericardium, myocardium, kidney, muscle, bone, testes, and dura. For this reason, the American College of Radiology recommends that prior to elective gadolinium-based MR contrast agent (GBMCA) administration, a recent (eg. past 6 weeks) glomerular filtration rate (GFR) assessment be obtained in patients with a history of iency who have 1. Renal disease (inchuding solitary kidney, renal transplant, renal tumor) 2. Age 260 years 3. History of hypertension 4. History of diabetes 5. History of severe hepatic disease/liver transplant/pending liver transplant: for these patients itis recommended that the patient's GFR assessment be nearly contemporaneous with the MR The incidence of NSF in patients with severe renal dysfunc tion (GFR <30) varies from 0.19 to 4%. A recent meta-analysis reported an odds ratio of 26.7 (95% Cl = 10,3-69.4) for develop- ment of NSF after gadolinium administration in patients with impaired renal function (GFR <30 mL/min/1.72 m). Thus, it is not recommended to administer gadolinium to any patient with a GRF below 30. Caution is advised for patients with a GRF below 45,

صفحه 66:
آقای 72 ساله با سابقه سر درد های راجعه با شکایت تورم پیشرونده اندام تحتانی مراجعه نموده است. در آزمایشات اولیه برای وی کرانینین ۵.۳ کشف شده است. 72-year-old male with progressive analgesic nephropathy. SICK (small, indented, and calcified kidneys) criteria was present. Image shows increased parenchymal thickness, coarse papillary calcifications bilaterally, and numerous indentations.

صفحه 67:
کدام یک از علل زیر در پاتوژنز نمای میکروسکوپیک مفابل در آقای ۴۳ ساله که با هماچوری و پروتینوری مراجعه نموده است نقش اساسی دارد؟؟ 63 ۲60۳۲9۳ Antiphospholipase a3 Ab Ig A deposits Hypocomplementemia Light & electron microscopic patterns of dense deposit disease

صفحه 68:
آقای ۲ ساله با درد فلانک چپ ‎Renal Tuberculosis‏ left autonephrectomy secondary to renal tuberculosis and ~@ probable cavitary tuberculosis of the left upper lobe was made. In countries in which index of suspicion for tuberculosis in patients with

صفحه 69:
38-year-old patient presented to the hospital with a 2-month history of fever, malaise,weight loss, and progressive _ weakness Bilateral Foot Drop in Polyarteritis Nodosa

صفحه 70:
5 ‏و‎ ‎a ‎4 ‎J ‎3 هاى مكرر ادرار

صفحه 71:
آقای ۵۴ ساله با سابقه بای پس کرونر ۷ روز قبل و درید اندام

صفحه 72:
A 37-year-old man was referred. Laboratory evaluation revealed a serum potassium level of 3.3 mmol per liter, a bicarbonate level of 16 mmol per liter, a calcium level of 9.3 mg per deciliter (2.3 mmol per liter), a phosphate level of 2.1 mg per deciliter (0.7 mmol per liter), a creatinine level of 3.0 mg per deciliter (265 mol per liter), a parathyroid hormone level of 62 Pg per milliliter, and an estimated lomerular filtration rate of 25 ml 1 6. ‏دأده م أكءاقع0 تامعط لطاع انها مأ ,5أكه ل زمی وهای دیا ۲625 دوع اق طز‎ 5 gni€aeatabed is limited to the renal medulla. Three years after sodium bicarbonate and potassium supplementation was restarted, the patient's renal function has remained stable.

صفحه 73:
A 44 y,o man with positive familial history for ESRD

صفحه 74:
Infectious Disease

صفحه 75:
Gram Negative Cell Wall a ‏لاقت ممعم‎ و 2 17 ۰ ۵ Sep Coban iad ‏مد ملسو‎ ‏«سس ل‎ Sep evil ‘pis opie Sete be ‏اس سس‎ ‏سس‎ ‏ایور‎ و سد سدم سس سس wa ‘Sep Talat ste te wate = Gram Positive Cell Wall, Sep, ae 3 اس مروت د حم ۳ ‎pune‏ مس روا Sepa Sat vee ‘aka compl + meine ‏سس دس‎ ‏تست‎ موه ‎Toys‏ ‎olin comgtxis‏ ی نمی لب هه تفر نت Sent ال مت رعاو عمج لس ی یز Gram Staining Procedure سید | سای سس ‎cae‏ Sep lz Begin witha | ( feed cells Step2: Flood slide swith crystal & ار مت Seep 5 Counter eta ‏که شا‎

صفحه 76:
a. Fever, lymphadenopathy, meningismus b. Carditis c. Ischemic bowel d. Cranial nerve palsies e. Arthritis The lesions in this patient developed 2 weeks after hiking in the woods of the Upper Midwest in April. If left untreated, this patient is at risk for development of all of the following except:

صفحه 77:
Lyme Disease Answer: c * Causative agent, Borrelia burgdorferi, is transmitted by Ixodes ticks ‘+ Highest incidence is during spring and summer * Tick must be attached more than 24 hours for transmission to occur + Stage 1: Within 30 days of infection, erythema chronicum migrans occurs in 80% of patients and may be associated with fever, lymphadenopathy, and meningismus. The rash resolves within 4 weeks * Stage 2: Onset is weeks to months after stage 1. Neurologic abnormalities occur in 10% to 15% of patients. Carditis occurs in 10% of patients * Stage 3: Begins months to years after infection. Arthritis, ‘occurs in 50% of untreated patients, becoming chronic in 10% to 20% + Enzyme-linked immunosorbent assay is positive for diag- nosis within 2 to 6 weeks * Early-stage disease may be treated with doxycycline, amox- | oF cefuroxime + Neurologic or cardiac involvement requires high-dose cef- triaxone or penicillin G

صفحه 78:
a. Fever | b. Positive blood cultures | Hypotension d. Involvement of 3 or more organ syster e. Erythroderma Clinical criteria for toxic shock syndrome include all of the following except:

صفحه 79:
Toxic Shock Syndrome Answer: b + Acute life-threatening illness caused by production of staphylococcal exotoxin (toxic shock syndrome toxin-1, TSS) + Centers for Disease Control and Prevention criteria: 6 required for confirmation; 5 of 6 indicate a probable case 1. Fever ‘Temperature of more than 38:9°C 2 Rash Diffuse, macular, and erythematous 3. Desquamation Especially of palms and soles 4. Hypotension In adults, systolic blood pressure less than 90 mm Hg 5. Multisystem Severe myalgias involvement Diarthea, vomiting, Liver dysfunction ‘Thrombocytopenia Renal insufficiency Mental status changes 6. Exclusion of other Negative results of blood, throat, causes cerebrospinal fluid cultures {usually not positive for Staphylococcus aureus) ‘+ Associations: menstruation with prolonged use of tampons, surgery (even if wound is not worrisome), bartier contracep- tives ‘+ With early and aggressive management and judicious use of antibiotics and supportive care, the mortality rate for toxic shock syndrome has been estimated at 3%. ‘The use of corticosteroids (to lessen the overwhelming inflammatory ‏]مرس رون‎ anid Yaamiricgobuilay eeniaine ‏م0050‎

صفحه 80:
خانم 30 ساله با شکایت تب و سر درد وترشحات ‎Soe‏ فراوان از بینی از 15 روز قبل که به درمان حمایتی جواب نداده است : توصیف و تشخیص؟ درمان پیشنهادی؟

صفحه 81:
GUIDELINES FOR THE DIAGNOSIS AND TREATMENT OF ACUTE ‘SINUSITIS IN ADULTS: Griteria__Treatment Recommendations* Moderate symptoms Initial therapy: eae Amoxicillin, 500 mg PO tid; or ‏ب‎ Amoxkillin/clavulanate, $00/125 mg PO tid or Severe srmptomsofary 875/125 mg PO bie duration, including uni- Penicillin allergy: lateral/focal facial swell- Doxycycline, 100 mg PO bid; or ing oF tooth pain ‘Clindamycin, 300 mg PO tid Exposure to antibiotics within 30 d or >30% prevalence of penicilin-resistant Streptococcus pneumoniae: ‘Amoxicillin/clavulanate (extended release), 200/125 mg PO bid; or ‘An antipneumococcal fluoroquinolone (eg, moxifloxacin, 400 mg PO daily) Recent treatment failure: ‘Amoxicillin/clavulanate (extended release), 2000 mg PO bid; or ‘An antipneumococcal fluoroquinolone (eg, moxifloxacin, 400 mg PO daily) “The duration of therapy is generally 7-10 days (with consideration ofa S-day course), with appropriate follow-up. Severe disease may warrant IV antibiotics and consideration of hospital admission. *Although the evidence is not as song, amoxiilin/clvulanate may be considered fr inal use, particularly focal rates of peniilin resistance or (lactamase production are high.

صفحه 82:
آقای 43 ساله مورد لنفوم غير هوچکین که 5 ماه قبل آخرین سیکل کموتراپی وی به اتمام رسیده است از 2 ماه قبل دچار ضایعات پوستی زیر و درد پیشرونده مچ پای راست شده است. نمونه گیری انجام شده را در زیر مشاهده میکنید. درمان پیشنهادی؟

صفحه 83:
9-4 Nocardia brasiliensis mycetoma. A. Draining sinuses and giant white grains with a seropurulent discharge. B. Radiography of the foot showing marked soft tissue enlargement and bony iytic lesions. € Direct microscopy of grains stained with Lugol's iodine (40) D. Periodic acid-Schiff stain of skin biopsy (X40). (Image provided by Roberto Arenas and Mahreen Ameen, St. John's Institute of Dermatology, Gi ys&

صفحه 84:
FIGURE 199-3 Nocardial abscesses in the right occipital lob: FIGURE 1991 Nocardial pneumonia. A dense infiltrate w! ible cavity and several nodules are

صفحه 85:
Nocardial pneumonia. ws bilateral nodules, wit

صفحه 86:
TREATMENT DURATION FOR NOCARDIOSIS Disease Duration Pulmonary or systemic Intact host defenses 6-12 months Deficient host defenses 12months* CNS disease 12 months! Cellulitis, lymphocutaneous 2 months syndrome Osteomyelitis, arthritis laryngitis, 4 months sinusitis Actinomycetoma 6-12 months after clinical cure Keratitis Topical: until apparent cure ‘Systemic: until 2-4 months after apparent cure ‘tn some patients with AIDS and CD44. lymphocyte counts of <200/uiL oF with chronic granulomatous disease, therapy for pulmonary or systemic disease must be continued Indefinitely. fall apparent CNS disease has been excised, the duration of therapy may be reduced to 6 months At the outset, 10-20 mg/kg of TMP and 50-100 mg/kg of SMX are given each day in two divided doses. Later, daily doses can be decreased to as little as 5 mg/kg and 25 mg/kg, respectively. In persons with sulfonamide allergies, desensitization usually allows continuation of therapy with these effective and inexpensive drugs.

صفحه 87:
a. Rifabutin b. Trimethoprim-sulfamethoxazole c. Pyrimethamine d. Fluconazole e. Acyclovir A37-year-old man with acquired immunodeficiency syndrome (AIDS) presents with fever, seizures, and altered mental status. His last known CD4 cell count was less than 50 cells/mm3. Which one of the following is the best prophylactic agent for this condition?

صفحه 88:
Central Nervous System Toxoplasmosis Answer: b * The preferred prophylactic agent for Pneumocystis carinii and Toxoplasma gondii is trimethoprim-sulfamethoxazole * If the patient is allergic to sulfonamides, pyrimethamine and dapsone are efficacious * Without prophylaxis, central nervous system toxoplasmosis will develop in 30% to 50% of patients with AIDS who are seropositive for Toxoplasma IgG * Magnetic resonance imaging is more sensitive than computed tomography for detecting central nervous system lesions of, toxoplasmosis * Typical appearance is of multiple ring-enhancing lesions within the brain parenchyma * Central nervous system lymphoma can mimic toxoplasma encephalitis, * Patients with a single lesion, who are seronegative, or who do not respond to antitoxoplasmosis therapy should be referred for brain biopsy * On biopsy, immunoperoxidase stain shows cysts and tachy- zoites of T. gondii

صفحه 89:
. Imipenem . Oxacillin and gentamicin Vancomycin and gentamicin |. Ceftriaxone and doxycycline . Amphotericin B ] ° In this case, in addition to diagnostic testing, which empiric therapy is most appropriate?

صفحه 90:
Sexually Transmitted Urethritis Answer: d * Urethral discharge with burning on urination and an itchy urethra are typical symptoms Neisseria gonorrhoeae: Gram-negative intracellular diplococcus Diagnosis by molecular gene probe, Gram stain, or culture Injectable ceftriaxone is first-line therapy Add doxycycline or azithromycin for treatment of Chlamydia trachomatis because 15% to 25% of patients with gonorrhea also have Chlamydia ‘Nongonoccocal urethritis: Symptomatically indistinguishable from gonococcal variety, but much more frequent Most common causes are C. trachomatis (25%-40%) and Ureaplasma urealyticura (40%-50%) Other causes include Mycoplasma -genitalium, Trichomonas vaginalis, and herpes simplex virus, + Sexual partners also should be treated and intercourse avoided until treatment is completed

صفحه 91:
1. What was the alternative medication? Ja. Pentamidine 1b. Pyrimethamine-sulfonamide Ie, Atoyaquone 1d. Dapsone Je. Clindamycin-primaquine 2, In addition to supplemental oxygen, which one of the following would be the most appropriate antidote? 2a. N-Acetyleysteine 2b. Amyl nitrite 2c. Methylene blue 2d. Deferoxamine 2e. 4-Methylpyrazole A patient with human immunodeficiency virus had a rash in association with trimethoprim-sulfamethoxazole used as pro- phylaxis for Pneumocystis carinii pneumonia. An alternative medication was used, but the patient took 3 times the normal dose by mistake. He became dyspneic, and headache, nausea and vomiting, and the skin discoloration shown here devel- oped (a normal hand is shown for contrast on the right).

صفحه 92:
Methemoglobinemia Due to Dapsone Overdose Answer: d Answer 2: ‘+ Methemoglobin is the ferric form of hemoglobin (HbFe>+) that cannot bind oxygen + Although sometimes inherited, it is more often formed by oxidative stress + Among drugs, nitrites are commonly associated with this condition + Chocolate-brown or slate-blue cyanosis unrelieved with oxygen suggests the diagnosis Arterial blood gas studies will show normal arterial oxygen pressure but low oxygen saturation. Pulse oximetry results ‘ill be inappropriately increased (the pulse oximeter result will be higher than the actual oxygen saturation, although the absolute value may still be below normal) * Methylene blue is the antidote for patients with severe hypoxia, unless they have glucose-6-phosphate dehyciro- genase deficiency, in which case it may worsen the clinical condition. If the patient has this deficiency, ascorbic acid can be used to reduce the methemoglobin ۰ N-Acetyleysteine is the antidote for acetaminophen overdose ‘+ Amyl nitrite is an antidote for cyanide overdose because nitrites precipitate the formation of methemoglobin, which isan alternative binder for cyanide, Thus, it would be con- traindicated in methemoglobinemia + Deferoxamine is the antidote for iron overdose + 4-Mothylpyrazole is an alternative to ethanol for the treat- ment of methanal toxicity

صفحه 93:
a. Conversion disorder b. Left brain abscess c. Seizure with Todd paralysis d. Left middle cerebral artery occlusion e. Atypical migraine A31-year-old man with Streptococcus viridans endocarditis has acute right hemiparesis and dysarthria. Which one of the fol- lowing is the most likely cause?

صفحه 94:
Left Middle Cerebral Artery Occlusion Due to Embolus in Infective Endocarditis Answer: d ‘+ Extracardiac manifestations of infective endocarditis includ the following: Embolic events Suppurative complications Immunologic reactions Osler’s nodes (which are painful) Roth’s spots Increased rheumatoid factor Glomerulonephritis ‘+ Systemic embolization occurs in 20% to 40% of patients wil left-sided endocarditis + Embolization may occur at any time, but risk decreases wit duration of treatment ‘Specifically vascular phenomena associated with infective endocarditis include the followin, Arterial emboli Mycotic aneurysms Intracranial hemorrhages Conjunctival hemorrhages Pulmonary infarcts (septic) Janeway lesions (hemorthagic lesions on the palms or soles, usually painless) Splinter hemorrhages

صفحه 95:
a. Magnetic resonance imaging of the head b. Computed tomography with thin cuts of the adrenal glands ‏ی‎ Small bowel biopsy with periodic acid-Schiff staining d. Dexamethasone suppression test e. Cosyntropin stimulation test ١ A 40-year-old man presents with hyperpigmentation, malab- sorptive diarrhea, weight loss, recurrent arthritis, and adenopa- thy. His wife is shown for contrast. Which one of the following tests is likely to confirm the diagnosis?

صفحه 96:
Whipple's Disease Answer: ¢ ‘+ Whipple's disease is a chronic relapsing, systemic infectious disease involving the central nervous system, heart, kidney: and small bowel » Tt occurs primarily in white middle-aged men + The causative agent is a gram-positive bacillus, Tropheryma whippelit + The name of the bacillus was chosen from the Greek word trophe, which means “nourishment,” thus highlighting the primary clinical feature of malabsorption ‘+ From 40% to 50% of patients have hyperpigmentation in ‘sun-exposed areas and scars + Other signs and symptoms include weight loss (80%), diarthea (75%), arthralgia (70%), lymphadenopathy (55%), abdominal tenderness (50%), fever (40%), edema (25%), Blossitis (20%), splenomegaly (10%), ascites (5%) ‘+ 90% of patients have steatorrhea on 72-hour stool studies + Anemia of chronic disease and thrombocytosis are common ‘Joint symptoms may precede intestinal manifestations by years + Diagnosis is established with small bowel biopsy showing periodic acid-Schiff-positive granules in macrophages with ‘gram-positive acid-fast bacillus-negative bacilli ‘+ Treatment is with trimethoprim-sulfamethoxazole for 1 year ‘+ Response to antibiotics is dramatic; most symptoms com- pletely resolve within 2 to 4 weeks ‘© Relapses are common (up to 40%)

صفحه 97:
. Blood cultures and empiric therapy with vancomycin, gentamicin, and metronidazole . Blood cultures, supportive care, lumbar puncture, and empiric doxycycline . Supportive care until serologic diagnosis can be made to direct therapy d. Discussion with family about withdrawal of support and palliative care . Transesophageal echocardiography 3 This 25-year-old man was hiking in North Carolina 1 week before development of rash and prostrating illness, including hypotension, meningoencephalitis, anemia, and bleeding. What is the most appropriate course of action?

صفحه 98:
Rocky Mountain Spotted Fever (RMSF) Answer: b * Although first described in Idaho in 1896, RMSF is most common in the mid-Atlantic states and Oklahoma, not the Rocky Mountains ۰ Rickettsia rickettsii, a gram-negative intracellular bacterium, is inoculated into humans by ticks (Dermacentor variabilis and D. anderson, the dog and wood tick, respectively) after at least 6 hours of feeding ۰ Most cases (>90%) occur between April and September * Incubation time is 2 to 14 days (mean, 7 days) ‘* Rash begins on the extremities and moves centrally ۰ Major symptoms and signs: fever (88%-100%), headache (79%-93%), myalgia (72%-92%), rash (74%-90%), known tick bite (54%-66%), nausea and vomiting (55%-60%), classic triad of fever, headache, and rash (45%-60%), classic tetrad of fever, headache, rash, and history of tick bite (3%-18%) * Serologic test result is not positive until 7 to 10 days after exposure; therefore treatment should not be delayed for diagnosis ‘+ Mortality rate is 20% to 25% in 8 to 15 days without appro- priate treatment and 5% despite appropriate treatment ‘+ Treatment of choice is doxycycline. Other tetracyclines and chloramphenicol are also effective

صفحه 99:

صفحه 100:
پسر 6 ساله با شکایت تب و خواب آلودگی و ضایعات پوستی زیر به اورژانس مراجعه نموده است. به گفته مادرش 3 روز قبل دچار ضایعات وزیکولار محدود در ناحيه صورت شده است که رفته رفته پیشرفت نموده است : 1 .توصیف ضایعات و تشخیص؟ 2.عامل بیماری و نام توکسین مربوطه؟ 3.درمان؟

صفحه 101:
1 ۱6۸۷۲۶ 172۰4 ۰ Evidence of staphylococcal scalded-skin syn- drome in a 6-year-old boy. Nikolsiy’s sign, with separation of the superficial laver of the outer epidermal laver. is visible, Reprinted with STAPHYLOCOCCAL SCALDED-SKIN SYNDROME SSSS primarily affects new- borns/and'children! The illness may vary from a localized blister to exfoliation of much/Of the’skin(Ssurface: The skin is usually fragile and often tender, with thin-walled, fluid-filled bullae. Gentle pres- sure results in rupture of the lesions, leaving denuded underlying skin (Nikolsky’s'sign; Fig. 172-4). Theimucous membranes are'usually spared. In more generalized infection, there are often constitutional symptoms, including fever, lethargy, and irritability with poor feeding. Significant amounts of fluid can be lost in more extensive cases. Illness usually follows localized infection at one of a number of possible sites. 5555 ‏كز‎ much less common among adults but can follow infections caused by exfoliative toxin=producing strains: THERAPY FOR OTHER TOXIN-MEDIATED DISEASES Therapy for staphylococcal food poisoning is entirely supportive. For SSSS, antistaphylococcal therapy targets the primary site of infection.

صفحه 102:
پسر 10 ساله که هفته قبل به علت تب و لرز و سرفه (گرافی زیر) در بخش اطفال بستری و با تشخیص پنومونی تحت درمان قرار گرفته است. 1 هفته بعد دچار ضایعات پوستی زیر به همراه درگیری مخاط دهان و بینی و چشم شده است. 1.عامل اولیه ایجاد پنومونی؟ 2.نام سندروم پوستی؟

صفحه 103:
Sevens Soesen oyndronns i child wth Mycoplatme ‏امتابم میم‎ (From Baum 5G. Mycoplasma preumoniae and atypial preumona. in Mandell GL. ETT Etreng DIAGNOSTICTESTS FOR RESPIRATORY MYCOPLASMA ‘PNEUMONIAE INFECTION Test Sensitivity, % Specificity, % Respiratory culture <60 100 Respiratory PCR 65-90 90-100 Serologic studies" 55-100 55-100 ‎ANTIMICROBIAL AGENTS OF CHOICE FOR MYCOPLASMA‏ -212عاقم1 ‎INFECTIONS:‏ ‎Drugis) ‎‘Azithromycin, danthromycin, erythromycin, ‎doxycycline, levofloxacin, moxifloxacin, ‎‘gemifionacin (nor ciprofloxacin or ofloxacin) ‎ ‎U.ureabyicum, U.parum ‎ ‎M hominis Doxycycline, clindamycin M.genitalium ‏متهم نمم‎ ‎‘Antimicrobial resistance has been reported in mycoplasmas, s described inthe text

صفحه 104:
limited, appropriate antimicrobial therapy significantly shortens the duration of clinical illness. Infection uncommonly results in. critical illness and only rarely in death. In some patients, long-term recur- rent wheezing or reactive airway disease may follow the resolution of acute pneumonia. The significance of chronic infection, especially as it relates to asthma, is an area of active investigation. Skin eruptions described with M. pneumoniae infection include ery- thematous (macular or maculopapular), vesicular, bullous, petechial, and urticarial rashes. In some reports, 17% of patients with M. pneumoniae pneumonia have had an exanthem. Erythema multiforme major (Stevens-Johnson syndrome) is the most clinically significant skin eruption associated with M. pneumoniae infection; it appears to occur more commonly with M, preurnoniae than with other infectious agents. ‘A wide spectrum of neurologic manifestations has been reported With M. pneumoniae infection. The most common are meningoen- cephalitis, encephalitis, Guillain-Barré syndrome, and aseptic menin- gitis. M. pneumoniae has been implicated as a likely etiologic agent in 5-7% of cases of encephalitis. Other neurologic manifestations may include cranial neuropathy, acute psychosis, cerebellar ataxia, acute demyelinating encephalomyelitis, cerebrovascular thromboembolic ‘events, and transverse myelitis. Hematologic manifestations of M. pneumoniae infection include hemolytic anemia, aplastic anemia, cold agglutinins, disseminated intravascular coagulation, and hypercoagulopathy. When anemia does ‘occur, it generally develops in the second or third week of illness. In addition, hepatitis, glomerulonephritis, pancreatitis, myocarditis, pericarditis, rhabdomyolysis, and arthritis (septic and reactive) have been convincingly ascribed to M. pneumoniae infection. Septic arthri- tis has been described most commonly in hypogammaglobulinemic patients. MYCOPLASMA PNEUMONIAE CLINICAL MANIFESTATIONS Upper Respiratory Tract infections and Pneumonia Acute M. pneumoniae infections generally manifest as pharyngitis, tracheobronchitis, reactive airway disease/ wheezing, ora nonspecific upper respiratory syndrome. Little evidence supports the commonly held belief that this organism isan important cause of otitis media, with or without bullous myringi- tis, Pneumonia develops in 3-13% of infected individuals; its onset is usually gradual, occurring over several days, but may be more abrupt. Although Mycoplasma pneumonia may begin with a sore throat, the ‘most common presenting symptom is cough. The cough is typically nonproductive, but some patients produce sputum. Headache, mal- aise, chills, and fever are noted in the majority of patients. ‘On physical examination, wheezes or rales are detected in ~80% of patients with M. pneumoniae pneumonia. In many patients, however, pneumonia can be diagnosed only by chest radiography. The most common radiographic pattern is that of peribronchial pneumonia with thickened bronchial markings, streaks of interstitial infiltration, and areas of subsegmental atelectasis. Segmental or lobar consolida- tion is not uncommon. While clinically evident pleural effusions are infrequent, lateral decubitus views reveal that up to 20% of patients have pleural effusions. ‘Overall, the clinical presentation of pneumonia in’ anv individual Patient is not useful for differentiating M. pneumoniae pneumonia from other types of community-acquired pneumonia, The possibility of M. pneumoniae infection deserves particular consideration when ‘community-acquired pneumonia fails to respond to treatment with a penicillin or a cephalosporin—antibiotics that are ineffective against mycoplasmas. Symptoms usually resolve within 2-3 weeks after the onset of illness. Although M. pneumoniae pneumonia is generally self-

صفحه 105:
آقای 4 ساله باغبان و دامدار که از 4 روز قبل دچار تب خفیف و ضایعه دردناک و خارش دار گسترش یابنده در انگشت 3و4 دست راست شده است که ابتدا حالت وزیکل داشته و سپس حالت پلاک پیدا کرده است

صفحه 106:
‘The gram-positive aerobic rod Erysipelothrix rhusiopathiae is most often associated with fish and domestic swine and causes cellulitis pri- marily in bone renderers and fishmongers. E. rhusiopathiae remains susceptible to most f-lactam antibiotics (including penicillin), eryth- romycin, clindamycin, tetracycline, and cephalosporins but is resistant to sulfonamides, chloramphenicol, and vancomycin. Its resistance to vancomycin, which is unusual among gram-positive bacteria, is of potential clinical significance since this agent is sometimes used in empirical therapy for skin infection. Fish food containing the water flea Daphnia is sometimes contaminated with M. marinum, which can cause cellulitis or granulomas on skin surfaces exposed to the water in aquariums or injured in swimming pools. Rifampin plus etham- butol has been an effective therapeutic combination in some cases, although no comprehensive studies have been undertaken. In addi- tion, some strains of M. marinum are susceptible to tetracycline or to trimethoprim-sulfamethoxazole. 00۳۳2۳ erysipeiod with ts characteristic purple, nonpurulent swelling of ‘the finger. Also known as whale finger or pork finger, this form of cellulitis caused by Erysipelothrix rhusiopathiae should not be confused with streptococcal or staphylococcal -erysipelas (see Fig. 441-4). (From Farrar WE, Wood MU, Innes JUA, Tubbs H. Infectious Diseases: Text ond Color Ata. 2nd ed. New York: Gower Medical Publishing: 1992)

صفحه 107:

صفحه 108:
Endocrinology : OSCE ۳

صفحه 109:
A 41-year-old woman presents with fatigue, anxiety, palpitations, and a tremor in her hands. She has lost approximately 9 ‏وها‎ in the past 4 months and reports feeling more comfortable in cool environments. Her menstrual cycle has become irregular, and her eyes are often irritated. The patient recalls that one of her grandmothers was briefly treated for a thyroid problem years ago. On physical examination, her pulse is 110 beats per minute and she is diaphoretic. She has eye irritation, with red conjunctivae, and has lid lag and proptosis in both eyes. She also has the appearance of staring and the reflexes characteristic of patients with hyperthyroidism. The remainder of the examination is normal. Laboratory testing reveals hyperthyroidism; the patient has a low level of thyrotropin and an elevated level of free thyroxine. A thyroid scan is ordered after the administration of radioactive (jodine-123) Which one of the following diagnoses is most likely in this patient? O Drug-induced thyroiditis O Graves’ disease Y¥ O Graves’ disease © Painful subacute thyroiditis O Toxic adenoma O Toxic multinodular goiter Tracer uptake was 65% within 6 hours after the administration of iodine-123. The scan revealed a homogeneous pattern of uptake.

صفحه 110:
The correct answer is Graves’ disease. Close [x] Graves’ disease is an autoimmune disorder caused by antibodies that bind to and activate G-protein—coupled thyrotropin receptors, stimulating the development of hypertrophy and hyperplasia in thyroid follicular cells. This stimulation causes both enlargement of the thyroid and an increase in the production of thyroid hormones. Graves’ disease is the most common cause of moderate-to-severe hyperthyroidism and is seen predominantly in women between the ages of 40 years and 60 years of age. Graves’ ophthalmopathy is clinically apparent in approximately 50% of patients with Graves’ disease. Patients have symptoms of hyperthyroidism, including weight loss, heat intolerance, difficulty sleeping, tremor, diarrhea, irritability and anxiety, and menstrual irregularity. Physical signs include tachycardia, goiter, resting tremor, hyperreflexia, and warm, moist, and smooth skin. In the eyes, signs include lid lag, proptosis, periorbital edema, conjunctivitis, erythema, and retraction of the upper eyelid. In rare instances, pretibial myxedema can also be detected.

صفحه 111:
A 25-year-old woman with no history of thyroid disease presents with a 5-day history of pain in her neck. She reports the development of a viral infection of the upper respiratory tract 1 week before presentation, with tendemess to palpation in her neck developing a few days later. She also reports having intermittent palpitations. On physical examination, she is febrile, with temperatures as high as 38.9°C, and her pulse is 105 beats per minute. She is also diaphoretic. There is no lid lag or appearance of staring. The thyroid gland is diffusely tender to palpation, and there is a slight resting tremor in her hands. Reflexes are brisk. The remainder of the examination is normal. Laboratory testing reveals hyperthyroidism; the patient has a low level of thyrotropin and an elevated level of free thyroxine. A thyroid scan is ordered after the administration of iodine-123. Which one of the following diagnoses is most likely in this patient? © Drug-induced thyroiditis O Graves’ disease O Painful subacute thyroiditis“ () Painful subacute thyroiditis O Toxic adenoma O Toxic multinodular goiter View cor No measurable tracer uptake had occurred 6 ‘hours after the administration iodine-123.

صفحه 112:
The correct answer is painful subacute thyroiditis. Close Painful subacute thyroiditis, also known as de Quervain's thyroiditis, is often but not necessarily preceded by viral infection of the upper respiratory tract. Typically, thyroiditis has three phases. The first phase is characterized by hyperthyroidism, which results from follicle destruction and the release of preformed thyroid hormone into the circulation. This phase often occurs before the clinical presentation. The second phase is ‏اه ای‎ hypothyroidism, which results from the loss of thyroid-hormone stores. In the third pha: re is resolution, with the return of euthyroidism (typically in 3 to 5 months). However, permanent hypothyroidism develops in approximately 5% of affected patients. Subacute thyroiditis is generally self-limited and is not treated with radioiodine or antithyroid drugs. Treatment addresses symptoms and is administered as needed, usualy in the form of a beta-adrenergic-receptor blocker.

صفحه 113:
‘A38-year-old woman with long-standing asymmetric enlargement of her neck (goiter) reports palpitations, a gradually increasing tremor in her hands, heat intolerance, and a loss of 4.5 kg over the past 6 months. She has no history of irradiation of the head and neck. Her grandmother also had a goiter. (On physical examination, her pulse is 108 beats per minute and her skin is moist and warm. She has mild lid lag but no appearance of staring. The mobile thyroid nodule on the left side of her neck measures, 5 cm in its largest diameter; there is no lymphadenopathy or bruit, and the nodule is nontender to palpation. Her hands show a slight tremor when at rest. Her reflexes are brisk, and the remainder of the examination is normal. A thyroid scan is ordered after the administration of jodine-123. The results of laboratory tests reveal hyperthyroidism. Which one of the following diagnoses is most likely in this patient? O Drug-induced thyroiditis, O Graves’ disease O Painful subacute thyroiditis 225 3 O Toxic adenoma CE Toxic adenoma 57 O Toxic multinodular goiter a ‏كفن‎ | Adele act indre-28 wis Toxic adenoma is a functional benign tumor of the thyroid that produces excessive Tracer uptake was 40% within 6 hours after amounts of thyroid hormone. the administration of iodine-123,

صفحه 114:
within 6 hours after the administration of ‘50-year-old woman who has had an asymmetric, enlarged goiter for many years reports the development of intermittent palpitations with an irregular heartbeat over the past 6 months. She has also lost 4.5 kg during this period, despite having a good appetite. On physical examination, her skin is warm and moist. She has three palpable, mobile thyroid nodules that measure between 2 cm and 4 cm in the largest diameter. There is no lymphadenopathy or bruit, and the thyroid is nontender to palpation. Her heart rate is elevated at 144 beats per minute; the rate is imegularly irregular, and a systolic murmur can be heard at the left lower sternal border. She has mild edema in both legs. Her reflexes are brisk, and the remainder of the examination is normal. Laboratory testing reveals hyperthyroidism; the patient has a low level of thyrotropin and an elevated level of free thyroxine. A thyroid scan is ordered after the administration of iodine-123. Which one of the following diagnoses is most likely in this patient? O Drug-induced thyroiditis O Graves’ disease © Painful subacute thyroiditis O Toxic adenoma O Toxic multinodular goiter _¥“ @) Toxic multinodular goiter — 1 3 ‘Several focal areas of increased radioiodine The correct answer is toxic multinodular goiter. ‏ددن‎ ‎۳7 0 uptake can be seen, with suppression of The prevalence of toxic multinodular goiter increases with age and is endemic in surrounding tissue. Tracer uptake was 28% iodine-123. regions with ahistory of iodine deficiency, such as Eastem Europe. Toxic multinodular adenomas may be treated with antithyroid drugs, radioiodine therapy, or thyroid surgery.

صفحه 115:
A 54-year-old woman presents with increased anxiety, diarrhea, sweating, and hand tremors that began 3 months earlier. She also lost 6.8 kg during this period. She has a history of atrial fibrillation and has been treated with ‘amiodarone for the past 2 years. She has no history of thyroid disease — thyroid function tests were normal before she began treatment with amiodarone. On physical examination, her skin is warm and moist. Her thyroid is normal and nontender and is without nodules. Her hands show a slight resting tremor. Her heart rate is 80 beats per minute and regular, and there is a systolic murmur at the left lower sternal border. She has mild edema in both legs. Her reflexes are brisk, and the remainder of the ‘examination is normal. Laboratory testing reveals hyperthyroidism; the patient has a low level of thyrotropin and an elevated level of free thyroxine. A thyroid scan is ordered after the administration of iodine-123. Which one of the following diagnoses is most likely in this patient? O Drug-induced thyroiditis ‏كي‎ © Drug-induced thyroiditis O Graves’ disease O Painful subacute thyroiditis O Toxic adenoma O Toxic multinodular goiter View commer Tracer uptake was low (<5%) within 6 hours after the administration of iodine-123.

صفحه 116:
The correct answer is drug-induced thyroiditis. Close Amiodarone-induced thyrotoxicosis occurs in up to 20% of patients receiving the drug. The condition may develop at any time during treatment and for up to 1 year after discontinuation of the drug. There are two types of amiodarone-induced thyroiditis. In type 1 disease, iodine induces an increase in thyroid-hormone synthesis and release. It is most common in patients with ahistory of subclinical thyroid disease and multinodular goiter. In type 2 disease, a destructive inflammatory process causes the release of preformed thyroid hormones and leads to thyroiditis. It is often challenging to distinguish the two forms of amiodarone- induced thyroiditis, since both are associated with low uptake of radioiodine. Some patients may have both types. On color-flow Doppler ultrasonography performed by an experienced ultrasonographer, type 1 disease reveals hypervascularity and type 2 disease reveals reduced blood flow. The primary treatment for type 1 disease is high doses of antithyroid drugs. Type 2 disease is treated with glucocorticoids.

صفحه 117:
آقای ۲۴ ساله با ضعف ,ء بی حالی و کاهش وزن به اورژانس شهدای تجریش مراجعه نموده است. سابقه هیچ گونه بیماری را نداشته و از ۲ هفته قبل دچار تهوع و استفراغ های مکرر شده که در آندوسکوپی وی به جز چندین کانون تیره رنگ در مخاط مری نکته ای نداشته است, در شرح حال بیمار کدام یک از یافته های زیر محتمل ترند؟؟ 8 درد مفاصل و میلژی ‎٠‏ پر نوشی و پرادراری 6 بی اشتهایی 6 تغییر رنگ پوست ‎٠‏ خوئريزى بينى ‎٠‏ كاهش تمايل به نمى © هييوتانسيون وضعيتى ‎٠‏ مصرف سيكار

صفحه 118:
کدام یک از یافته های آزمایشگاهی در اين بیمار محتمل است؟ + > هیپرناترمی ۵.هپرکلمی ADH Gli. ‏پلاسما‎ ۲5۲۷ ‏سطح‎ ase ‏©.لنفوسيتوز‎ ‏©.ايوزينوفيلى‎ .آلكالوز متابوليىك

صفحه 119:
کدام یک از, تست های زیر در تایید تشخیص در این مرحله اولویت بیشتری دارد؟؟ ۱.الکترولیت های سرم و ©86© چم ۸.سطح آلدوسترون سرم .يه ۲.فعالیت رنین پلاسما 4و ٩.سطح‏ سرمی اسید های چرب با زنجیره متوسط ۳.کورتیزول ادرار ۲۴ ساعته ۰ تست های عملکرد کلبه يم ‎chu!‏ آدرنوکورتیکرتروپین سرم #ي» ۱ تست اينفيوزن سالين ‎A 2. TSH ‏۶.تست کوزینتروپین ۳ ‏۷.تست سرکوب دگزامتازون با ۱ میلیگرم

صفحه 120:
رم أزمايشكاه موكزى باتوبيولوزى ۷ | ا" ‎sige =‏ 3 ‎tay‏ )2 سید مجتی نگوقدممطق ۳۳

صفحه 121:
]۳7۳[ ۶۱6۱/5 ۸۱۸۵ 5۷۱۸۵۲۵۱۷۸5 ۵۴ ۸۵۸6۱۷۸۸ ۱۸5۵۴۴۱۵۵۵۷6۲ ISURE#06-15 Clinical features of Addison's disease. Note the hyperpiamentation in areas of increased friction including (A) palmar creases (8) dorsal foot, (C nipples and allay region, and () patchy hyperpigmentaten ofthe oral mucosa Signs and Symptoms Caused by Glucocorticoid Deficiency TB>Fatigue, lack of energy Weight loss, anorexia ‎pain‏ مامز ,جاورالا ‎Fever ‎Normochromic anemia, lymphocytosis, eosinophilia ‎Slightly increased TSH (due to loss of feedback inhibition of TSH release) ‎Hypoglycemia (more frequent in children) ‎Low blood pressure, postural hypotension ‎Hyponatremia (due to loss of feedback inhibition of AVP release) ‎‘Signs and Symptoms Caused by Mineralocorticoid Deficiency (Primary ‎‘Adrenal insufficiency Only) ‎“Abominal pain, nausea, vomiting ‎Diziness postural hypotension ‎Salt caving ‎Low blood pressure, postural hypotension ‎Increased serum creatinine (due to volume depletion) Hyponatremia ‎Hyperkalemia ‎Signs and Symptoms Caused by Adrenal Androgen Deficiency Lack of energy ‎Dry and itchy skin ¢n women) ‎Loss of ibido (in women) ‎Loss of allay and pubic hair (in women) ‎Other Signs and Symptoms ‎Hyperplamentation primary adrenal insufciency oni) (due to excess of proopiomelanacortn [POMC]-derived peptides) ‎Alabaster-colored pale skin (secondary adrenal insufficiency only) (due to deficency of POMC.denved peptides) ‎

صفحه 122:
با توجه به اقدامات قبلی و نتایج به دست آمده در حال حاضر کدام تست تشخیصی را پیشنهاد میکنید: ‎Adrenal PET scan‏ Brain CT + Contrast CXR Jo Plasma VLCFA JS 17-OH progestron JS Adrenal CT w/wo ‏سای‎ Whole body bone scan Check mutation of VHL gene

صفحه 123:
مدالیته تصویر برداری- توصیف ضایعه- تشخیص X-linked adrenoleukodystrophy has an incidence of 1:20,000 maleg and is caused by mutations in the X-ALD gene encoding the feroxfsnrllhembrane transporter protein ABCD1; its disruption results in accumulation of very long chain (>24 carbon atoms) fatty acids. Approximately 50% of cases manifest in early childhood with rapidly progressive white matter disease (cerebral ALD); 35% present during adolescence or in early adulthood with neurologic features indicative of myelin and peripheral nervous system involvement (adrenomyeloneuropathy [AMN]). In the remaining 15%, adrenal insufficiency is the sole manifestation of disease. Of note, distinct mutations manifest with variable penetrance and phenotypes within affected families.

صفحه 124:
کدام یک از درمان های زیر را در این مرحله پیشنهاد میکنید : ۱.تزریق هفتگی متیل پردنیزولون ۲۰ میلیگرم ۲.قرص هیدروکورتیزون ۱۵ میلیگرم صبح و ۱۰ میلیگرم شب ‎JS‏ ‏۳.امپول دگزامتازون ۸ میلیگرم ماهانه ۴.قرص پردنیزولون ۲۵ میلیگرم روزانه ۵.قرص فلودروکورتیزون ۱۰۰ میکروگرم روزانه 4

صفحه 125:
ا ا اك ب 1 ۳ [0 agnosis + Pama corse 39-80 min stor 239 ‏انه قل جاجد چیه ون‎ (Corso post ‏رای‎ 500 001 + OBC, sarum cei, potactum, cearine, ura, TSH ‘ileal agnosia Plana ACTH, ls ‏عمد اه و‎ 7[ ماد لمعيه سما ‎ow toro ACTH, ema i‏ و ‎Store) tera! arene)‏ 1 1 0 سجاوه ‎‘elcome cco‏ ‎(wet rtany 0‏ نیمه ‎reste I‏ ‎Hse dt exponen‏ ‎Senator?‏ Hypothalamic | | Elon of bod ture? ‏امعم وم‎ | | Color lo ACTH ‘tency

صفحه 126:
‎A‏ ۱ سا حملااةا سن ذرة: وتران قل وار :یاف هلى زير را توصيف نموده و تشخيص احتمالى خود را بنويسيد : ‎ ‎ ‎ ‎ ‎The VHL gene (among other genes) encodes an E3 ubiquitin ligase that regulates expression of hypoxia-inducible factor 1. Loss of VHL is associated with increased expression of vascular endothelial growth factor (VEGF), which induces angiogenesis. Although the VHL gene can be inactivated by all types of mutations, patients with pheochro- mocytoma predominantly have missense mutations. About 20-30% of patients with VHL have pheochromocytomas, but in some families the incidence can reach 90%. ‎ ‎ ‎ ‎

صفحه 127:
Von Hippel-Lindau disease. A. Retinal angioma. All subsequent panels show findings on MRI B-D. Hemangioblastomas of, the cerebellum (8) in brainstem (Q and spinal cord (D).E. Bilateral pheochromocytomas and bilateral renal clear cell carcinomas F, Multiple pancreatic cysts. (Parts A and D from HPH Neumann et al: Adv Nephrol Necker Hosp 27:361, 1997, © Elsevier, Part B from SH Morgan, JP Grunfeld feds}:

صفحه 128:

صفحه 129:
آقای ۲۹ ساله با درد سینه ناگهانی» تعریق وتهوع از چند ساعت قبل به اورژانس مراجعه نموده است. در بدو ورود نوار قلب زیر از بیمار اخذ شده است. ۱ ۱

صفحه 130:
سوالات حاشیه ای که ممکن است پرسیده شود: توصیف دقيق و تشخيص؟؟؟ ۲ اختلال الكتروليتى كه ممكن است الكوى نوارى مشابه ايجاد نمايند؟ كدام خانواده آنتى اريتميك قادر است باعث 57718 كردد؟ كدام سندروم ها باعث 5711 منحصرا در ليد هاى 1/1 تا3لا ميكردد؟ [۲۱۲31۰3 DIFFERENTIAL DIAGNOSIS OF ST-SEGMENT ELEVATIONS Ischemia/myocardial infarction Noninfarction, transmural ischemia Prinzmetals angina, and probably Tako-tsubo syndrome, which may alo exactly simulate classical acute infarction) Acute myocardial infarction Postmyocardial infarction (ventricular aneurysm pattern) Acute pericarditis Normal variants (including benign ‘early repolarization’ pattems) Left ventricular hypertrophy/left bundle branch block? Other (aren) Acute pulmonary embolism? Brugada pattems (right bundle branch block-lke pattern with ST elevations In right precordial leads) Class 1C antiarthythic drugst DC cardioversion Hypercalcemia? Hypetkalemiae Hypothermia ( [Osborn] waves) Nonischemic myocardial injury Myocarditis Tumor invading left ventricle Trauma to ventricles Usually localized ‏لاه را‎ Source: Modified ror AL Goldberger etal Goluberger Clinical Electrocardioaraghy: A ‘Simplified Approach, 8 ed. Philadelphia lsevier/Saunders, 2013.

صفحه 131:
کدام یک از موارد زیر در اخذ شرح حال و تعيين پلان درمانی اين بیمار در اولویت است؟؟؟ 0 0 0© 0 © © © سابقه فاميلى از نظر آترواسكلروز زودرس مصرف سيكار ابى زود هاى قبلى درد مینه مشابه حملات مکرر درد شکم از کودکی سابقه بيماريهاى روماتولوزيك در کودکی مصرف تفريحى الكل شدت درد عفونت ویرال اخیر مجاری هوایی فوقانی © سابقه مسرف كوكيين ‎CO)‏ واكسيناسيون دوره كودكى © سابقه خونريزى مغزى ‏قشار خون بالا ‏© سابقه سکته مغزی اخیر © داروهاى مصرفى بيمار ©6 زمان دقيق شروع درد

صفحه 132:
در قدم بعد کدام یک از اقدامات زیر جهت بیمار ضرورت بیشتری دارد؟؟ 0 9 0 00 © © © © شروع فیبریتولیتیک در صورت عدم کنتر اندیکاسیون تست ورزش نوار قلب سریال چک آنزیم های قلبی معاینه دقیق قلبی-عروقی-ریوی ارسال نمونه ادرار اسکن هسته ای قلب پایدار ساختن راه هوایی اکوکاردیوگرافی اورژانسی ()) ارسال آزمایشات اوليه و بروفايل جربى © لس اكسيمترى 0 شروع ليدوكايين وريدى جهت جلوكيرى از أريتمى () شروع أسبيرين و بلاويكس © حفظ اشباع اكسيزن شريانى در حد ۹۸ درصد ‎OC‏ سی‌تی آنژیوگرافی عروق کرونر )) بررسی بیمار از نظر دیسکسیون أيورت 0 مانیتورینگ قلبی (2) نتروگلیسرین زیرزبانی هر ۰ دقیقه تا ۲ ساعت

صفحه 133:
نتایج آزمایشات بیمار به شرح زیر میباشد : LDL:349 HDL: 32 Cholest: 490 14 ELECTROLYTES: NI CPK:232 MB:55 Tn: 12 ALT:29 AST:54 BILI:1.1 BUN: 21 Cr:1.1 ESR:12 CRP:11 CBC: NI PT,PTT: NI

صفحه 134:
کدام یک از تشخیص های زیر جهت بیمار بیشتر مطرح است : هیپرتریگلیسریدیمی فامیلیال دیس بتالیپوپروتینمی شیلومیکرونمی سیتواسترولمی هیپرلیپیدمی فامیلیال ترکیبی

صفحه 135:
: ‏یک از یافته های زیر در معاینه بالینی اين بیمار مورد انتظار است‎ plas Oe vA

صفحه 136:
Physical signs of heterozygous familial hypercholesterolemia, as a result of cholesterol deposition within macrophages in specific _ sites. Tendinous xanthomas, for example, manifests first as thickening of, and later as deposits within, extensor tendons. A: lateral borders of thickened Achilles’ tendons are shown with arrows. B: tendinous xanthomas can also occur in the extensor tendons of the hands,

صفحه 137:
کدام یک از خانواده های دارویی زير در بیماران مبتلا به اين اختلال کاریردی ندارد؟ ‎١‏ .استاتين ها 00 ‎٩‏ ازتیماب 00 ۲فیرات ها ‎as. So‏ 5 ۴.رژیم غذایی حاوى سو راي 00 LDL ‏آفرز‎ .4 ‎FFP dust?‏ هيه ‎۱3 ‏وی‎ inhibitors ‎Ginhibitirs of MTP

صفحه 138:
Patients with homozygous FH must be treated aggressively to delay the onset and progression of CVD.(Receptor defective patients sometimes respond to statins and other LDL-lowering drug classes such as a cholesterol absorption inhibitor or a bile acid sequestrant, which upregulate the LDL receptor activity. Two drugs that reduce the hepatic production of VLDL and thus LDL,a small-molecule inhibitor of the microsomal TG transfer protein (MTP) and an antisense oligo- nucleotide to apoB, are approved in the United States for the treatment of adults with homozygous FH and can be considered. PCSK9 inhibi- tors, which work through increasing LDL receptor availability, appear to have some benefit in receptor-defective patients and are under clini- cal development. LDL apheresis is used to lower plasma LDL levels in these patients and can promote regression of xanthomas as well as slow the progression of atherosclerosis. Because the liver is quantitatively the most important tissue for removing circulating LDLs via the LDL receptor, liver transplantation is effective in decreasing plasma LDL-C levels in this disorder but is infrequently used because of the associated problems with immunosuppression.

صفحه 139:
آقای ۵۵ ساله دیابتی جهت بررسی تاری دید پیشرونده مراجعه نموده است توصیف ۳ يافته مهم- تشخیص-درمان The most effective therapy for diabetic retinopathy is preven- tion. Intensive glycemic and blood pressure control will delay the development or slow the progression of retinopathy in individuals With either type 1 or type 2 DM. Paradoxically, during the first 6~12 months of improved glycemic control, established diabetic retinop- athy may transiently worsen. Fortunately, this progression is tempo- rary, and in the long term, improved glycemic control is associated With less diabetic retinopathy. Individuals with known retinopathy may be candidates for prophylactic laser photocoagulation when initiating intensive therapy. Once advanced retinopathy is present, improved glycemic control imparts less benefit, although adequate ‘ophthalmologic care can prevent most blindness. Regular, comprehensive eye examinations are essential for all individuals with DM (see Table 418-1). Most diabetic eye disease ‘can be successfully treated if detected early. Routine, nondilated eye examinations by the primary care provider or diabetes special- ist are inadequate to detect diabetic eye disease, which requires an ophthalmologist for optimal care of these disorders. Laser photo- ‘coagulation is very successful in preserving vision. Proliferative reti- nopathy is usually treated with panretinal laser photocoagulation, whereas macular edema is treated with focal laser photocoagula- tion and anti-vascular endothelial growth factor therapy (ocular injection). Aspirin therapy (650 mg/d) does not appear to influence the natural history of diabetic retinopathy.

صفحه 140:
A 23-year-old woman was admitted with a 3-day history of fever, cough productive of blood-tinged sputum, confusion, and orthostasis. Past medical history included type 1 DM. Physical examination indicated postural hypotension, tachycardia, and rapid respiration. Examination of the thorax lobreptrectese] consolidatié¢vin the righPitwer lobe. * توصیف کامل اختلال اسیدو باز بیمار * محتملترین تشخیص * تست درخواستی بعدی و درمان طبق الگوریتم should be lic acidosis. dilution of with type 1 increase in several ‘The diagnosis of DKA is usually not challenging but considered in all patients with an elevated AG and metabol Hyperglycemia and ketonemia (positive acetoacetate at a 1:8 or greater) are sufficient criteria for diagnosis in patients diabetes mellitus. The A[HCO,”] should approximate the the plasma AG (AAG), but this equality can be modified by factors. For example, the AAG will often decrease with IV hydration, as o the urine, glomerular filtration increases and ketones are excreted meq/L meq/L mea/L ‏امهم‎ ‎mg/dL ‎9/4 ‎ma/dL mmHg mmHg ‏امهم‎ Sodium 13 Potassium 50 Chloride 96 02 14 Blood urea nitrogen (BUN) 20 Creatinine 13 Glucose 450 Arterial Blood Gases On Room Air pH 739 Poo, 24 Pao, 89 (HCO; 14 Urinalysis Urine ketones Positive 4+ Glucose Positive 4+ Sarum Ketones Stronaly positive 1:8

صفحه 141:
۲۳۱۱۱۹۱۲۵۰ ۱۸۸۱۸۸۹۸۸۵۱۸۲ OF DIABETIC KETOACIDOSIS 1. Confirm hagrosis(T plasma glucose positive serum letones, metabalic eciosi). 2. Admit ta hospital intensive care sting maybe necessary fr frequent monitoring oF pH <7. 00 or uncanscous ‏ترچ‎ ‎Serum slecilytes (Nat, Mg, Cr ieatbonat, phosphate) ‏یف‎ status —pH, HCO, Peo, Prhyroxyburyrate Renal function (creatinine urine ouput 4. Replace fis: 2-2 L of 096 saline over fist 1-3 h (10-20 ml/g per hour), suosequently, 045% saline at 250-500 mL/h; change to 5% alu- cose red 043% sane at 150-250 ml/h when plasma glucose reaches 250 mo/a. (139 TOI 5. Administer shortacting insulin 1V (01 units), then 0 unitsfig per hour by continuous Vnfusion nctease wot tnecold if no resorse by 2th the nel serum potas s-<33 mol (33 mec). do not admin nsdn unt the possum ie camected 6. Assess patient What prectated the episode (noncomesiance, infection trauma, pregnancy, infarction, cocare) inate aproprave workup fr ] evert (cultures, C3R, ECG) 7, Measure caoilry glucose ever 1-2 measure electrolytes (espectly W bicabonate phosphate] and anion gap every 4h fest 28h Monitor blood pressure, pulse resptatlons, ment status, id intake and ouiputevery 1-4 9, Replace 6: 10 mea/h when plasma K* <5.0-52:meayl (or 20-30 mea/ of fusion Mui), ECG norma, une flow and normal cestinine ocr mmenved: administer 40-20 mec/h vihen lesa K"<35 meey OF Diearbonate fs gwen. inal eum potachmn 98.2 mcd. (52 mer) do not supplement K" Unt the pass i corected 10. See ex about bicarbonate or phosphate supplementation 11, Continue abore until patient stable, lucose goals 22-139 mmol. (050-250 mg/L), and acidosis resolved. sun infusion may be decreased 16 095-01 unlsig per hout 12: Adiiniste long-acting insulin as scan as patient seating. Alow fra oa hour overap i insulin infusion and SC insulin injection

صفحه 142:
‘A41-year-old woman presents with fatigue, anxéety, palpitations, and a tremor in her hands. She has lost approximately ‘9kg in the past 4 months and reports feeling more comfortable in coo! environments. Her menstrual cycle has become irregular, ard her ayes are oftan irritated. The patiant recalls that one of her grandmothers was briafy treated for 2 thyroid problem years ago. ‘On physical examination, her pulse is 1/0 beets per minute and she is dicphoretic. She has eye initation, with red cconjunctivae, and has lid lag and proptosis in both eyes. She also has the appearance of staring and the refexes characteristic of patents with hyperhyroidism Tha remainder of the examination is normal. Laboratory testing reveals hyperthyroidism; the patient has 2 low level of thyrotropin and an elevated level of free thyroxine. A thyroid scan is ordered after the adminsstration of radioactive (iodine-123). Which one of the following diagnoses is most likely in this patient? O Drug-inducad thyroicitis, CO Graves’ disease © Painful subacute thyroiditis (O Toxic adenoma O Toxic multinodular goiter Tracer uptake was 65% within 6 hours after ۰ the administration of iodine-123. The scan V O Graves’ disease revealed a homogeneous patter of uptake.

صفحه 143:
A25-year-old woman with no history of thyroid disease presents with a 5-day history of pain in her neck. She reports the development of a viral infection of the upper respiratory tract 1 week before presentation, with tendemess to palpation in her neck developing a few days later. She also reports having intermittent palpitations. ‘On physical examination, she is febrile, with temperatures as high as 38.9°C, and her pulse is 105 beats per minute. She is also diaphoretic. There is no lid lag or appearance of staring. The thyroid gland is diffusely tender to palpation, and there is a slight resting tremor in her hands. Reflexes are brisk. The remainder of the examination is normal. Labor testing reveals hyperthyroidism; the patient has a low level of thyrotropin and an elevated level of free thyroxine, A thyroid ‘scan is ordered after the administration of iodine-123. Which one of the following diagnoses is most likely in this patient? Drug-induced thyroiditis CO Graves’ disease © Painful subacute thyroiditis O Toxic adenoma O Toxic multinodular goiter Y © Painful subacute thyroiditis | Romesnuratie aces tpl ae coon

صفحه 144:
۸ 3 ‏0امعو‎ ۱۱۵9۵0 ۷/۳ long-standing asymmetric enlargement of her neck (goiter) reports palpitations, a gradually increasing tremor in her hands, heat intolerance, and a loss of 4.5 kg over the past 6 months. She has no history of ‘radiation of the head and neck. Her grandmother also had a goter. On physical examination, her pulse is 108 beats per minute and her skin is moist and warm. She has mild lid lag but no appearance of staring. The mobile thyroid nodule on the left side of her neck measures, 5 cm in its largest diameter; there ig no lymphadenopathy or bruit, and the nodule is nontender to palpation. Her hands chow a slight ramor when at rest. Hor reflexes aro brisk, and the remainder of the examination ie normal. A thyroid soan is orderad afior the administration of odine-123. The results of laboratory tests reveal hyperthyroidism. Which one af the follwing diagnoses is most likely in this patient? O brugsinduced thyroiitis O Graves’ disease © Painful subacute thyroiditis O Tox adenoma © ‘oxic multinodular goiter Adiscrete area of iodine-123 uptake is ۱ ‘noted, with no uptake in surrounding tissue. Y O Toxic adenoma Tracer uptake was 40% within 6 hours after the administration of iodine-123,

صفحه 145:
١ ‘A50-year-ald woman who has had an asymmetric, enlarged goiter for many years reports the development of intermittent palpitations with an regular heartbeat over the past 6 months. She has also lost 4.5 kg during this period, ‘On physical oxarination, her skin is warm and mist. She has three palpablo, motile thyroid nedulos that measure between 2 cm and 4 cm nthe largest diameter. There is no lymphadenopathy or bruit, and the thyroid is nontender to palpation. Hor heart rata is clovated at 144 boats por minuto; tho rata is irregularly iroguler, and a systole murmur can bbe heard at the let jower sternal border. She has mild edema in both lees. Her reflexes are brisk, and the remainder of the examination is normal. Laboratory testing reveals hyperthyroidism; the patient hes a low level of thyratrapin and en eevated level of free thyroxine. A thyroid scan is ordered after the administration of iocine-125. daspite having a good appatte. ‎Which one of the following diagnoses is most likely in this patient?‏ م ‎ ‎Several focal areas of increased radioiodine uptake can be seen, with suppression of surrounding tissue. Tracer uptake was 28% within 6 hours after the administration of jodine-123. ‎© Drug-induced thyroiditis O Graves’ disease ‎O Painful subacute thyroiditis O Toric adenoma ‎Toxic multinodular goiter ‎ ‎ ‎Y @ Toxic multinodular goiter ‎ ‎

صفحه 146:
54-year-old woman presents with increased anxiety, diahea, sweating, and hand tremors thal began 3 months earlier. She also lost 6.8 kg during this period. She has a history of atrial flrilation and has been treated with amiodarone for the past 2 years. She has no history of thyroid disease — thyroid functon tests were normal before she began ireatment with amiodarone On physical examination, her skin is warm and moist. Her thyroid is normal and nontender and is without nodules. Her hands show a slight resting tremor. Her heart rate is 80 beats per minute and regular, and there is a systolic murmur at the left lower sternal border. She has mid edema in both legs. Her reflexas are brisk. and the remainder of the examination is normal. Laboratory testing reveals hyperthyroidism; the patient has a low level of thyrotropin and an. olovated lovel of free thyroxine. A thyroid coan is orderod after the administration of iodine-123, Which one of the following diagnoses is most likely in this patient? O Drug-induced thyroiditis, O Gaves' disease ‏ره‎ ‎O Toxic adenoma Toxic multinodular goiter Roll over scan for more information ‘Tracer uptake was low (<5%) within 6 hours VY @ Drug-induced thyroiditis | after the Sa metas

صفحه 147:
آقای ۴۳ ساله با پلی اوری-پلی دیپسی: ۱.نام ساختار های مشخص شده را ؟. با توجه به شكل كدام يك از علل ‎١‏ ‏زير جهت توجيه علايم وى محتمل تر است: رای a ۱۰۸۱6۵۳۲۵96016 DI 1 2 پلی دیپسی اولیه ,۲ Y.Central Dl

صفحه 148:
Urinary frequency, nocturia, enuresis 24+h urine volume and osmolarity on unrestricted fd intake Volume <a0 mig Osmolarity >800 mosml تست ‎[Aeon]‏ سس ¥ ¥ ‎Primary polyaipsia Pituitary Di‏ FIGURE4044 Simplified approach to the differential diagnosis Of diabetes insipidus. When symptoms suggest ciabetes insipidus (Dd the syndrome shoul be differentiated from a genitourinary (U) atmormalty by measuring the 24h urine vakime and osmolarity on Unrestricted fluid irate If D's confirmed, basal plasma arginine vaso- pressin (AVE) should ba measured on unrestricted uid intake. f AVP is normal or elevated (>1 pg/ml, the patient probably has nephrogenic Di. However. if plasma AVP is low or undetectable, the patent has ether pituitary Dl ox primary polydipsia,

صفحه 149:
خانم ۵۵ ساله با تنگی نفس پیشرونده احساس خواب آلودگی شدید طی روز : نام يافته ها (در مجموع ۵ مورد) - تشخیص؟؟؟ HEEL PAD THICKENING 5 ‏و اما درمان انتخابى در صورت‎ Lateral radiograph of skull reveals enlarged sella with ‏شرایط بالینی مناسب؟؟؟‎ double flooring, thickened skull vault, pneumosinus dilatans and prognathism.

صفحه 150:
However, 50% of patients require at least 8 years for GH levels to be suppressed to <5 j1g/1; this level of GH reduction is achieved in about 90% of patients after 18 years but represents suboptimal GH suppression. Patients may require interim medical therapy for several years before attaining maximal radiation benefits. Most patients also experience hypothalamic-pituitary damage, leading to gonadotro- pin, ACTH, and/or TSH deficiency within 10 years of therapy. In summary, surgery is the preferred primary treatment for GH-secreting microadenomas (Fig. 403-5). The high frequency of GH hypersecretion after macroadenoma resection usually necessitates adjuvant or primary medical therapy for these larger tumors. Patients unable to receive or respond to unimodal medical treatment may benefit from combined treatments, or can be offered radiation. Git RECEPTOR ANTAGONIST Pegvisomant antagonizes endogenous GH action by blocking peripheral GH binding to its receptor. Consequently, serum IGF-I levels are suppressed, reducing the deleterious effects of excess endogenous GH. Pegvisomant is administered by daily subcutane- ‘us injection (10-20 mg) and normalizes IGF-I in ~70% of patients. GH levels, however, remain elevated as the drug does not target the pituitary adenoma. Side effects include reversible liver enzyme elevation, lipodystrophy, and injection site pain. Tumor size should be monitored by MRI. Combined treatment with monthly somatostatin analogues and weekly or biweekly pegvisomant injections has been used effec- tively in resistant patients. DOPAMINE AGONISTS Bromocriptine and cabergoline may modestly suppress GH secre- tion in some patients. Very high doses of bromocriptine (220 mg/d) or cabergoline (0.5 mg/d) are usually required to achieve modest GH therapeutic efficacy. Combined treatment with octreotide and cabergoline may induce additive biochemical control compared with either drug alone. RADIATION External radiation therapy or high-energy stereotactic techniques are used as adjuvant therapy for acromegaly. An advantage of radiation is that patient compliance with long-term treatment is not required. Tumor mass is reduced, and GH levels are attenuated over time.

صفحه 151:
خانم ۳۵ ساله دونده با پیوست مزمن و شکستگی فمور هر دو سمت “Salt-and-pepper” skull produced by alternating zones of lucency and sclerosis. Loss of trabecular detail in the diploic space blurs t the:separation between i inner. and outer tubules ‏مس افا‎ Parameter Guldetine

صفحه 152:
پسر ۲۳ ساله سیگاری و دیابتی با سابقه مثبت خانوادگی بیماری کرونر با حملات مکرر درد شکم. نام یافته ها و درمان های انتخابی؟؟

صفحه 153:
FAMILIAL HYPERTRIGLYCERIDEMIA (FHTG) FHTG is characterized by ele- vated fasting TGs without a clear secondary cause, average to below average LDL-C levels, low HDL-C levels, and a family history of hypertriglyceridemia. Plasma LDL-C levels are often reduced due to defective conversion of TG-rich particles to LDL. In contrast to FCHL, apoB levels are not elevated. The identification of other first-degree relatives with hypertriglyceridemia is useful in making the diagnosis. Unlike in FCHL, this condition is not generally associated with a sig- nificantly increased risk of CHD. However, if the hypertriglyceridemia is exacerbated by environmental factors, medical conditions, or drugs, the TGs can rise to a level at which acute pancreatitis is a risk. Indeed, management of patients with this condition is mostly geared toward reduction of TGs to prevent pancreatitis. Individuals with this phenotype generally have reduced lipolysis of TRLs, although overproduction of VLDL by the liver can also con- tribute. No single gene has been identified in which mutations cause this disorder, whereas combinations of gene variants have been shown to cause this phenotype. A more appropriate term for this condition might be polygenic hypertriglyceridemia.

صفحه 154:
دختر ۲۱ ساله با آنمی و اختلالات خلقی و عدم تعادل: توصیف دقیق مدالیته و یافته ها- تشخیص patients with WD (a) T2W axial image showing ntense signal changes in bilateral putamen and thalami; (b) TLW axial sequence with hyperintensity of both pallidum (c) FLAIR axial sequence revealing midbrain tectal plate signal

صفحه 155:
توصیف یافته ها (۲ مورد) 1۹۴ Macronodular liver cirrhosis, relative sparing of the caudate lobe, and moderate splenomegaly.

صفحه 156:
Kayser-Fleischer ring 3 يافته؟؟؟؟! ! !! شرمنده! فقط محض خنده عکس اسکلرای نرمال بود

صفحه 157:
Kayser-Fleischer (K-F) rings seen in Wilson's disease is due to copper deposition in the Descemet's membrane in the sclero-corneal junction. K-F rings seen here on blue iris before and after treatment.

صفحه 158:
پسر ۱۱ ساله با سردرد» تاری دید و پلی اوری از ۳ ماه قبل 0۳555 A benign suprasellar tumour arising from squamous epithelial remnants of Rathke's pouch (from which the anterior pituitary develops) * It is the most common paediatric tumour of the suprasellar cistern Visual failure (due to optic chiasm compression) ® headache (secondary to raised intra- cerebral pressure following obstruction of the foramen of Monro) » endocrine disturbances (due to pituitary compression) * Although it occurs most frequently during childhood, a further peak is seen during the 6"" decade ‘The tumour tends not ‏ما‎ expand the pituitary fossa unless it is very large (which is a dif- ferentiating feature from a pituitary macroadenoma) Location It is commonly located within the hypotha- lamic region » it less commonly involves both the suprasellar and intrasellar regions ® a purely intrasellar region is rare CT A cystic and calcified suprasellar tumour (calcifica~ Craniopharyngiomas in two children. (A.B) The first child has a Hon #8 often. present in childhood tumours but less ‏یه‎ large suprasellar, prepontine and middle cranial fossa tumour 9 doen an cial furacurs) which is causing considerable mass effect on the brainstem and is 25۳ Although usually cystic, there can also be a ¢olid ۱ . .: or mixed cystic/solid appearance ® solid. components encasing the basilar artery (arrowheads). There are calcified ‏اقب بو‎ components (arrows). The cystic components are of higher density © Cystic components: TIWE: slightly hyp on CT in keeping with proteinacous contents. (due to the protein content) » T2WI

صفحه 159:
خانم 55 ساله دیابتی, تشخیص ؟ Necrobiosis lipoidica diabeticorum is an uncommon dis- order, accompanying diabetes in predominantly young women. This usually begins in the pretibial region as an erythematous plaque or papules that gradually enlarge, darken, and develop irregular margins, with atrophic centers and central ulceration.

صفحه 160:
خانم ۶۵ ساله با سر درد و تاری دید پیشرونده : توصیف؟ تشخیص؟ درمان انتخابی؟ 7 ۱ ‏ما‎ Anterior Posterior

صفحه 161:
PHARMACOLOGIC AGENTS APPROVED FOR TREATMENT OF PAGET'S DISEASE Dose and Mode Name of Delivery Normalization of ALP Zoledronic acid 5 ‏وم‎ V over 15 min 90% of patients at 6 mo Pamidronate 30mg IV/d over4hon ~50% of patients 3 days Risedronate 30mg PO/dfor2mo 73% of patients Alendronate 40 mg PO/d for 6 mo 63% of patients Tiludronate 800 mg PO daily for 35% of patients 3mo Etidronate 200-400 mg PO/d x 15% of patients 6mo Calcitonin (Miacalcin) 100 U SC daily for 6-18 (Reduction of ALP by up mo (may reduce to50U to 50%) 3 x per wk)

صفحه 162:
خانم 27 ساله با 2 نوبت حمله پانکراتیت در 6 ماه گذشته مراجعه نموده است

صفحه 163:
* Diaguvsis: ۰ ‏بمزروجو)‎ MO lipol vic uctivity iat postkeparia ‏لام‎ ‏هون وا‎ dePiciewy, if ‏اه تاه‎ the oddiiza oF cored plaiswu. * Oolevular sequeuviay oP the yours. و * Oetry Pat resiriciiva (IS qd) wi ‏امه متمموان طلطیاموه<‎ * Pick vile * Ve the wute seticg oP ‏0-4141)صمه‎ dePiciewy: PRC ۰ ) yeve therupy upprowk: dipoyeur fiparvover

صفحه 164:

صفحه 165:
ی )] « ‏عومرمز()‎ ‎* Oorapectaiog oP very high levels oP ‏تسا مج‎ *“B-quaciPicaiva” by utrarecrPugation (OLOL-C/TC >O.90) * Lipoproteis electrophoresis (broud B bac) * Ourtear waaetiz ‏باس مسا‎ * OO@-bused wethods (apoE yeutppicn): uprx~PC/EO * Dreakvet " biPestle * Clivicaiog or teeukoeat oP previpitaicg Poctors * Grotics — اا اراد بر ۰

صفحه 166:
Miscellaneous

صفحه 167:
. Use of HLA-identical grafts eliminates the risk of this con- dition . This condition typically occurs 7 to 21 days after transplan- tation . Increased age is a risk factor for this condition . Sex mismatch (female donor, male recipient) is a risk factor for this condition . Values on liver function tests are often increased in this con- dition 37-year-old man received a bone marrow transplant 2 weeks previously for acute myelogenous leukemia. Two days ago, itching, diarrhea, pain and numbness in his palms and soles, and the skin lesions shown here developed. Each of the follow- ing statements about this condition is true except:

صفحه 168:
Acute Graft-Versus-Host Disease Answer a + Occurs 7 to 21 days after transplantation + Donor T cells attack host HLA antigens ‘+ Mean frequency in adults with HLA-identical grafts is 35% ۰ Risk factors include the following: Increased age HLA mismatch Sex mismatch (female donor, male recipient) Irradiation Suboptimal immunosuppression + Alfects the skin, gastrointestinal tract, and liver + Pruritus and pain may be the first sensations, commonly followed by an erythematous measles-like maculopapular exanthem. Acral erythematous lesions may develop on, palms, soles, and ears ‘+ Blistering and exfoliation are common ‘+ Digestive tract involvement may present (from less severe to more) as nausea, vomiting, anorexia, diarrhea, malabsorption, abdominal pain, ileus, and ascites ‘+ Values on liver function tests are often increased; jaundice and hepatomegaly may develop ‘+ Treatment involves intensifying immunosuppression, such as high-dose corticosteroids, cyclosporine, and cyclophos- phamide ‘© Severe cases (grade IV) have a high mortality rate (>80%)

صفحه 169:
a. Ehlers-Danlos syndrome b. Cutis laxa c. Osteogenesis imperfecta d. Congenital contractural arachnodactyly e. Marfan syndrome ‘These are the hand and skin findings of a 26-year-old woman who complains of early satiety. Her past medical history is notable for upper gastrointestinal bleeding and rectal prolapse. ‘What is the diagnosis?

صفحه 170:
Ehlers-Danlos Syndrome Answer: a Ehlers-Danlos syndrome is characterized by highly elastic connective tissue Many forms (up to 15) of Ehlers-Danlos syndrome exist The autosomal-dominant forms of the disease account for 90% of reported cases Patients have hyperextensible and lax joints that are prone to dislocation Patients with skin manifestations have hyperextensible, fragile skin that heals poorly, characteristically forming wide, thin, “fish-mouth” scars. The skin may have a velvety texture Patients are predisposed to the following: Gastrointestinal motility disorders Visceral diverticulosis Mitral valve prolapse (up to 50% of patients) Dilatation of the aortic root Pes planus Scoliosis Degenerative arthritis Pneumothorax Dilatation of the pulmonary artery Angina

صفحه 171:
عتانطاح5 . b. Cystic medial necrosis c. Rheumatoid arthritis d. Hypertension . Atherosclerosis p ° A 54-year-old woman presents with nonreproducible upper back discomfort and dysphagia. She has a history of coronary artery disease and abdominal aortic aneurysm repair 6 years ago. Thoracic computed tomography scans are shown. In the United States, which one of the following risk factors is most commonly associated with this condition?

صفحه 172:
Descending Thoracic Aortic Aneurysm Answer: e ۰ Aortic aneurysms are classified by location (abdominal or thoracic) and shape (fusiform or saccular, as in this case) * Thoracic aneurysms are further classified as ascending or descending, based on their location proximal or distal to the aortic arch * Most often aortic aneurysms are asymptomatic * Compression of adjacent structures (the esophagus in this, case) may result in symptoms. Leakage of blood from the aneurysm may cause acute pain and may be a sign of impending rupture * Acute rupture without warning is the most common cause of symptoms. Therefore, clinical suspicion for the diagnosis should be high + When one aneurysm is detected, the patient should be screened for the presence of other occult aneurysms * Size correlates with risk of rupture, but not as exactly as for abdominal aortic aneurysms. Nonetheless, most authors favor surgical management for thoracic aortic aneurysms that are more than 6 cm in diameter * Atherosclerosis is the most commonly associated condition. Others include hypertension, giant cell arteritis, syphilis, Ehlers-Danlos syndrome, rheumatoid arthritis, trauma, cystic medial necrosis, and Marfan syndrome

صفحه 173:
a. Exposure to sunlight b. Diffuse muscular pain c. Proctoscopic examination d. Fat aspirate biopsy e. Hypertension 57-year-old man with known lymphoproliferative disorder presents with gastrointestinal bleeding and diarrhea. Periorbital purpura may occur in this patient and is associated with which one of the following?

صفحه 174:
Amyloidosis With Pulmonary and Gastrointestinal Involvement Answer: ¢ ‘+ Amyloidosis may have multiorgan involvement, including liver, kidney, gastrointestinal tract, and heart. Up to 90% of, patients with primary amyloidosis have cardiac dysfunction + The liver may become infiltrated in up to 25% of cases and indicates extensive involvement and a poor prognosis, ‘+ Manifestations result from deposition of an amorphous, insoluble protein-polysaccharide complex in the tissue ‘+ There are several types of amyloid, and their classification is based on the type of protein fibrillar deposition: Type AA is associated with reactive systemic amyloidosis and is found in hereditary ot acquired chronic inflam- matory disease Type AL is associated with systemic amyloidosis and is found in multiple myeloma and monoclonal gam- mopathies| ‘* Gastrointestinal manifestations include diarrhea, megacolon, and fecal incontinence ‘+ Periorbital purpura may occur after proctoscopic examination ‘+ Fat aspiration is confirmatory of the diagnosis in up to 80% of patients. Rectal biopsy confirms the diagnosis in up to 75% of patients ‘+ Congo red stain of histologic specimens may reveal a char- acteristic apple-green birefringence under crossed polarized light. This remains the standard test for diagnosis

صفحه 175:
a. Liver b. Kidneys c. Heart d. Lungs e. Pancreas The nails shown here are those of a 56-year-old ill-appearing woman. Which one of the following organ systems has the hicheet Hkohhand of heine diceaced?

صفحه 176:
Yellow Nail Syndrome Answer: d * Classic triad: Yellow nails Extremity lymphedema Pleural effusions + Nail findings: Discoloration: yellow to yellow-green or brownish yellow Slow growth Onycholysis + Pulmonary associations (not in 100%): Pleural effusion (35%-40%; lymphocyte predominant, often bilateral, one-third are recurrent) Restrictive and obstructive lung defects Bronchiectasis (20%) * Systemic associations (not in 100%): Rhinosinusitis Chronic edema of lower extremities Breast edema Raynaud’s phenomenon Pericardial effusion + Lymphatic insufficiency is common and results in the edema * Nail changes may be reversible and do not necessarily cor relate with other manifestations of the syndrome (such as pulmonary disease, edema) © No treatment has been proved effective

صفحه 177:
a. Systemic lupus erythematosus b. Defective porphyrin metabolism cc. Autoimmune reaction to self-antigens in the skin d. Autosomal recessive defect in DNA excision repair mechanism. e. A vertically transmitted hepatitis virus In this 15-year-old girl, these lesions developed in sun-exposed. areas. She is exquisitely sensitive to sunburn. What is the cause of this rare disorder?

صفحه 178:
Xeroderma Pigmentosum Answer: d ‘+ Autosomal recessive disorder found in all racial groups ‘+ Multiple mutations have been identified, but all involve defects in the excision repair mechanism of damaged DNA in skin exposed to ultraviolet light (nucleotide excision repair) ‘Skin findings: Actinic keratoses Cutaneous melanoma Progressive atrophy Irregular pigmentation Telangiectases Basal cell carcinoma Squamous cell carcinoma ‘+ Patients are very sensitive to sun exposure, as evidenced by reports of tongue tumors ‘Skin cancers develop at an early age (median, 8 years) ‘+ Most patients die of these malignancies by the third decade of life ‘+ Risk for skin cancer is 2,000- to 10,000-fold greater in patients with xeroderma pigmentosum than in age-matched controls ‘* Ophthalmic and neurologic disorders also occur * Diagnosis is established with the fibroblast survival test after exposure to ultraviolet light

Pulmonology: OSCE & PMP خانم 48ساله که 3ساعت پس از سقوط از پله منزل دچار تگی نفس و هذیان گویی شده و در بررسی های اولیه شواهد باال به دست آمده است؟ CASE 1: A 65 y,o man with a history of 40 p/y smoking presented because of 4 Ws of progressive exertional dyspnea and non productive cough!!! CASE 3 : a 67 y,oman with progressive dyspnea since 2 y ago CASE 4 : A 40 y,o nonsmoker woman with progressive dyspnea of 4 m duration accompanied with fever, elevated LDH and ANA CASE 5 : A 85 y,o man with acute progressive shortness of breath since 48 h ago CASE 6 : A 81 Y,O MAN WITH FLU-LIKE SYMPTOMES SINCE 3 WEEKS AGO + EXERTIONAL DYSPNEA Low- and high-power microscopic views of COP. Note the absence of old fibrosis or architectural distortion. The high-power view shows typical branching appearance & granulation tissue plugs and the mild interstitial inflammatory infiltrate. something that is هر دو بیمار آقای 57ساله سیگاری p/y 30با شکایت سرفه و تنگی نفس از 5ماه قبل توصیف و تشخیص؟ درمان؟؟ بدون شرح!!! CASE 2: خانم جوان با شکایت درد قفسه سینه از صبح روز مراجعه با انتشار به بازوی راست و تنگی نفس. بیمار سابقه فشار خون و GERDو سرفه مزمن را نیز داشته است. نام دو دستگاه مشاهده شده را بگویید و برای هر مورد یک مثال دارویی بزنید: آقای 45ساله با تنگی نفس و هموپتزی مراجعه نموده است .با توجه به تگرافی بیمار 5 تشخیص افتراقی مطرح نمایید: CAVITY mnemonic : C = cancer bronchogenic carcinoma : most frequently SCC cavitatory metastasis : again most frequently SCC A = autoimmune granulomas Wegener's granulomatosis rheumatoid arthritis (rheumatoid nodules) V = vascular pulmonary embolus / pulmonary infarction I = infection pulmonary abscess TB, fungal, staph aureus T = trauma Pneumatocoeles Y = youth CPAM pulmonary sequestration bronchogenic cyst خانم 33ساله با شکایت هموپتزی مراجعه نموده است. محتملترین تشخیص با توجه به گرافی بیمار؟ خانم 44ساله با شکایت تنگی نفس مراجعه نموده و به علت رویت پلورال افیوژن تحت آسپیراسیون مایع پلور قرار گرفته است .تب نداشته و مایع بدبو نمیباشد. تشخیص؟ 3علت اصلی ایجاد این حالت؟ Nephrology: OSCE & PMP آقای ۵۷ساله با شکایت تیره شدن رنگ ادرار و ادم پیش رونده اندام تحتانی .در معاینه اولیه فشار خون باال داشته و معاینات قلب وپوست وشکم طبیعی است .سمع ریه رال منتشر دارد .بررسی های اولیه به شرح زیر است: : یافته مه>م آن را نام ببرید۳ نوار را تفسیر نموده و MAT: The cardinal features are irregularity and a plethora of different Pwave morphologies. You need to have •Tachycardia (HR >100) •Irregular rate •Variability in P wave morphology and PR length تفسیر و تشخیص: هماچور>ی + پر>وتینور>ی + گلوکوزاوی+ پیور>ی ‏Urine Dipsteak: Urinary Sediment: RBC Cast Granular Cast WBC Cast تفسیر: :تفسیر Diffuse pulmonary haemorrhage with Bilateral GGO :تفسیر P-ANCA against تفسیر: چه درمانی در این مرحله برای بیمار پیشنهاد میکنید؟؟ A 12 y.o girl with sudden onset of generalized edema and heavy proteinuria پسر ۲۳ساله با نارسایی پیشرونده کلیه وهماچوری و شرح حال مثبت خانوادگی از نظر ESRD پسر ۱۵ساله با هماچوری میکروسکوپی پایدار و پروتینوری خفیف هر کی بگه جایزه داره! خانم 52ساله با چندین نوبت بررسی جهت هماچوری میکروسکوپی : .1توصیف دقیق رادیولوژی .2تشخیص .3سه مورد از اختالالت آزمایشگاهی محتمل .4درمان آقای 54ساله مورد ESRDتحت دیالیز با سابقه بستری 2ماه قبل به علت سر درد های راجعه شدید در یک بیمارستان تخصصی اعصاب که از 2 هفته قبل دچار ضایعات پیشرونده پوستی شده. .1تشخیص .2علت .3توصیه های پیشگیرانه احتمالی در. ساله با سابقه سر درد های راجعه با شکایت تورم پیشرونده اندام تحتانی مراجعه نموده است72 آقای . کشف شده است۵.۳ آزمایشات اولیه برای وی کراتینین 72-year-old male with progressive analgesic nephropathy. SICK (small, indented, and calcified kidneys) criteria was present. Image shows increased parenchymal thickness, coarse papillary calcifications bilaterally, and numerous indentations. کدام یک از علل زیر در پاتوژنز نمای ساله۴۳ میکروسکوپیک مفابل در آقای که با هماچوری و پروتینور>ی مراجعه نموده است نقش اساسی دارد؟؟ C3 nephritic factor Antiphospholipase a3 Ab Ig A deposits Hypocomplementemia Light & electron microscopic patterns of dense deposit disease ساله با درد فالنک چپ۷۲ آقای left autonephrectomy secondary to renal tuberculosis and probable cavitary tuberculosis of the left upper lobe was made. In countries in which tuberculosis is endemic, it is important to have a high index of suspicion for tuberculosis in patients with 38-year-old patient presented to the hospital with a 2-month history of fever, malaise,weight loss, and progressive weakness آقای ۵۴ساله با عفونت های مکرر ادراری؟؟ آقای ۵۴ساله با سابقه بای پس کرونر ۷ر>وز قبل و در>د اندام A 37-year-old man was referred. Laboratory evaluation revealed a serum potassium level of 3.3 mmol per liter, a bicarbonate level of 16 mmol per liter, a calcium level of 9.3 mg per deciliter (2.3 mmol per liter), a phosphate level of 2.1 mg per deciliter (0.7 mmol per liter), a creatinine level of 3.0 mg per deciliter (265 μmol per liter), a parathyroid hormone level of 62 pg per milliliter, and an estimated glomerular filtration rate of 25 ml Type 1 distal acidosis, in which nephrocalcinosis is per minute perrenal 1.73 tubular m2 of bodypresentarea but is limited to the renal medulla. Three years after surface sodium bicarbonate and potassium supplementation was restarted, the patient's renal function has remained stable. A 44 y,o man with positive familial history for ESRD Infectious Disease Endocrinology : OSCE & PMP کدام یک از یافته های آزمایشگاهی در این بیمار محتمل است؟ .1هیپرناترمی .2هیپرکالمی .3افزایش ADH پالسما .4کاهش سطح TSH .5لنفوسیتوز .6ایوزینوفیلی .۷آلکالوز متابولیک کدام یک از> تست های زیر در تایید تشخیص در این مرحله اولویت بیشتری دارد؟؟ .۱الکترولیت های سرم و CBC .۸سطح آلدوسترون سرم .۲فعالیت رنین پالسما .۹سطح سرمی اسید های چرب با زنجیره متوسط .۳کورتیزول ادرار ۲۴ساعته .۱۰تست های عملکرد کلیه .۴سطح آدرنوکورتیکوتروپین سرم .۱۱تست اینفیوژن سالین ۵. TSH .۶تست کوزینتروپین .۷تست سرکوب دگزامتازون با ۱میلیگرم Adrenal autoantibodies Neg :با توجه به اقدامات قبلی و نتایج به دست آمده در حال حاضر کدام تست تشخیصی را پیشنهاد میکنید • Adrenal PET scan • Brain CT + Contrast • CXR • Plasma VLCFA • 17-OH progestron • Adrenal CT w/wo contrast • Whole body bone scan • Check mutation of VHL gene تشخیص؟؟؟- توصیف ضایعه-مدالیته تصویر برداری • Noncontrasted MRI - FLAIR Sequence • Diffuse bilateral white matter lesion with posterior predominance • XLALD کدام یک از درمان های زیر را در این مرحله پیشنهاد میکنید : .۱تزریق هفتگی متیل پردنیزولون ۳۰میلیگرم .۲قرص هیدروکورتیزون ۱۵میلیگرم صبح و ۱۰میلیگرم شب .۳امپول دگزامتازون ۸میلیگرم ماهانه .۴قرص پردنیزولون ۲۵میلیگرم روزانه .۵قرص فلودروکورتیزون ۱۰۰میکروگرم روز>انه خانم ۲۳ساله با حمالت سر درد و تپش قلب و تاری دید :یافته های زیر را توصیف نموده و تشخیص احتمالی خود را بنویسید : حد اقل ۲نوع تومور شناخته شده دیگر در این بیماران را نام ببرید؟؟؟ آقای ۲۹ساله با درد سینه ناگهانی ،تعریق وتهوع از چند ساعت قبل به اورژانس مراجعه نموده است .در بدو ورود نوار قلب زیر از بیمار اخذ شده است. • سواالت حاشیه ای که ممکن است پرسیده شود: • توصیف دقیق و تشخیص؟؟؟ • ۲اختالل الکترولیتی که ممکن است الگوی نواری مشابه ایجاد نمایند؟ • کدام خانواده آنتی اریتمیک قادر است باعث STEگردد؟ • کدام سندروم ها باعث STEمنحصرا در لید های V1تاV3 میگردد؟ کدام یک از موارد زیر در اخذ شرح حال و تعیین پالن درمانی این بیمار در اولویت است؟؟؟ سابقه فامیلی از نظر آترواسکلروز زودرس سابقه مصرف کوکایین مصرف سیگار واکسیناسیون دوره کودکی اپی زود های قبلی درد سینه مشابه سابقه خونریزی مغزی حمالت مکرر درد شکم از کودکی سابقه دیابت سابقه بیماریهای روماتولوژیک در کودکی قشار خون باال مصرف تفریحی الکل سابقه سکته مغزی اخیر شدت درد داروهای مصرفی بیمار عفونت ویرال اخیر مجاری هوایی فوقانی زمان دقیق شروع درد در قدم بعد کدام یک از اقدامات زیر جهت بیمار ضرورت بیشتری دارد؟؟ شروع فیبرینولیتیک در صورت عدم کنترااندیکاسیون ارسال آزمایشات اولیه و پروفایل چربی تست ورزش پالس اکسیمتری نوار قلب سلایر شروع لیدوکایین وریدی جهت جلوگیری از آریتمی چک آنزیم های قلبی شروع آسپیرین و پالویکس معاینه دقیق قلبی-عروقی-ریوی حفظ اشباع اکسیژن شریانی در حد ۹۸درصد ارسال نمونه ادرار سی تی آنژیوگرافی عروق کرونر اسکن هسته ای قلب بررسی بیمار از نظر دیسکسیون آیورت پایدار ساختن راه هوایی مانیتورینگ قلبی اکوکاردیوگرافی اورژانسی نیتروگلیسرین زیرزبانی هر ۲۰دقیقه تا ۲ساعت کدام یک از تشخیص های زیر جهت بیمار بیشتر مطرح است : هیپرتریگلیسریدیمی فامیلیال هیپرکلسترولمی فامیلیال دیس بتالیپوپروتینمی شیلومیکرونمی سیتواسترولمی هیپرلیپیدمی فامیلیال ترکیبی کدام یک از یافته های زیر در معاینه بالینی این بیمار مورد انتظار است : Physical signs of heterozygous familial hypercholesterolemia, as a result of cholesterol deposition within macrophages in specific sites. Tendinous xanthomas, for example, manifests first as thickening of, and later as deposits within, extensor tendons. A: lateral borders of thickened Achilles' tendons are shown with arrows. B: tendinous xanthomas can also occur in the extensor tendons of the hands, کدام یک از خانواده های دارویی زیر در بیماران مبتال به این اختالل کاربردی ندارد؟ .۱استاتین ها .۹ازتیماب .۲فیبرات ها .۱۰کلستیرامین .۳پالسمافرز .۴رژیم غذایی حاوی MCFA .۵آفرز LDL .۶تزریق FFP ۷. PCSK9 inhibitors ۸. Inhibitirs of MTP آقای ۵۵ساله دیابتی جهت بررسی تاری دید پیشرونده مراجعه نموده است توصیف ۳یافته مهم -تشخیص-درمان? A 23-year-old woman was admitted with a 3-day history of fever, cough productive of blood-tinged sputum, confusion, and orthostasis. Past medical history included type 1 DM. Physical examination indicated postural hypotension, tachycardia, and rapid respiration. Examination of the thorax suggested consolidation in the right lower lobe. • توصیف کامل اختالل اسیدو باز بیمار • محتملترین تشخیص • تست درخواستی بعدی و درمان طبق الگوریتم آقای ۴۳ساله با پلی اوری-پلی دیپسی: .۱نام ساختار های مشخص شده ر>ا بنویسید: .۲با توجه به شکل کدام یک از علل زیر جهت توجیه عالیم وی محتمل تر> است: ۱.Nephrogenic DI پلی دیپسی اولیه ۲. ۳.Central DI : ساله با تنگی نفس پیشر>ونده احساس خواب آلودگی شدید طی ر>وز۵۵ خانم تشخیص؟؟؟- ) مورد۵ نام یافته ها (در مجموع HEEL PAD THICKENING Lateral radiograph of skull reveals enlarged sella with double flooring, thickened skull vault, pneumosinus dilatans and prognathism. و اما درمان انتخابی در صورت شرایط بالینی مناسب؟؟؟ ساله دونده با یبوست مزمن و شکستگی فمور هر دو سمت۳۵ خانم “Salt-and-pepper” skull produced by alternating zones of lucency and sclerosis. Loss of trabecular detail in the diploic space blurs انتخاب the separation شرط برای۴ between inner and outer درمان جراحی؟؟؟ tubules پسر> ۲۳ساله سیگاری و دیابتی با سابقه مثبت خانوادگی بیماری کرونر با حمالت مکر>ر درد شکم .نام یافته ها و درمان های انتخابی؟؟ تشخیص- توصیف دقیق مدالیته و یافته ها: ساله با آنمی و اختالالت خلقی و عدم تعادل۲۱ دختر احتمالی؟؟ MRI in patients with WD (a) T2W axial image showing hyperintense signal changes in bilateral putamen and thalami; (b) T1W axial sequence with hyperintensity of both pallidum; and (c) FLAIR axial sequence revealing midbrain tectal plate signal مورد) ؟؟؟۲( توصیف یافته ها Macronodular liver cirrhosis, relative sparing of the caudate lobe, and moderate splenomegaly. نام یافته؟؟؟؟!!!! شرمنده! فقط محض خنده عکس اسکلرای نرمال بود Kayser-Fleischer (K-F) rings seen in Wilson's disease is due to copper deposition in the Descemet's membrane in the sclero-corneal junction. K-F rings seen here on blue iris before and after treatment. پسر ۱۱ساله با سردرد ،تاری دید و پلی اوری از ۳ماه قبل خانم 55ساله دیابتی .تشخیص ؟ خانم ۶۵ساله با سر درد و تاری دید پیشرونده : توصیف؟ تشخیص؟ درمان انتخابی؟ خانم 27ساله با 2نوبت حمله پانکراتیت در 6ماه گذشته مراجعه نموده است. توصیف و نام یافته ها؟ تشخیص؟ Familial chylomicronemia syndrome ▪ Diagnosis: ▪ Assaying TG lipolytic activity in postheparin plasma. ▪ In apoC-II deficiency, it normalizes after the addition of normal plasma. ▪ Molecular sequencing of the genes. ▪ Treatment: ▪ Dietary fat restriction (15 g/d) with fat-soluble vitamin supplementation. ▪ Fish oils ▪ In the acute setting of apoC-II deficiency: FFP ▪ A gene therapy approach: alipogene tiparvovec Familial dysbetalipoproteinemia ▪ Diagnosis: ▪ Documentation of very high levels of remnant lipoproteins “β-quantification” by ultracentrifugation (VLDL-C/TG >0.30) ▪ Lipoprotein electrophoresis (broad β band) ▪ Nuclear magnetic resonance lipoprotein profiling ▪ ▪ DNA-based methods (apoE genotyping): apoE2/E2 ▪ Treatment: ▪ Lifestyle ▪ Elimination or treatment of precipitating factors ▪ Statins ▪ Cholesterol absorption inhibitors, fibrates, niacin Miscellaneous

180,000 تومان